<<

P a g e | 1

Q1. The "Black Hole Tragedy" was associated with a) British conquest of b) Third Anglo-Afghan War c) British conquest of d) First Anglo-Afghan War

Q2. In the 18th century, due to which of the following reasons Bengal made unprecedented progress? 1. There were no inter-border disputes in this region. 2. There was a relatively stable political condition. 3. No external invasion was experienced by this region. Select the correct answer using the code given below: a) 3 only b) 1 and 2 only c) 1 and 3 only d) 1, 2 and 3

Q3. With reference to the Treaty of , consider the following statements: Under this treaty 1. Mughal Emperor Shah Alam II agreed to issue a farman granting the diwani of Bengal, and Orissa to the East Company. 2. Nawab Shuja-ud-Daula agreed to surrender Allahabad and Kara to the . Which of the statements given above is/are correct? a) 1 only b) 2 only c) Both 1 and 2 d) Neither 1 nor 2

Q4. Which of the following factors can be attributed to the success of the British in India? 1. Fair selection system 2. Superior arms and military strategy 3. Better military discipline 4. Support of second-line leaders 5. Strong financial backup Select the correct answer using the codes given below: a) 1, 2 and 5 only b) 2, 3 and 5 only c) 1 and 4 only d) 1, 2, 3, 4 and 5

Q5. Consider the following statements: 1. Gobind Singh transformed the into a militant sect in defence of their religion and liberties. 2. Ranjit Singh organized the Sikhs into 12 misl. Which of the statement(s) given above is/are correct? a) 1 only

©Jatin Verma All Rights Reserved. https://www.jatinverma.org P a g e | 2

b) 2 only c) Both 1 and 2 d) Neither 1 nor 2

Q6. With reference to the treaty of Salbai, consider the following statements: 1. It provided the British twenty years of peace with the Marathas. 2. It enabled the British to exert pressure on Mysore with the help of the Marathas in recovering their territories from Haider Ali. Which of the statements given above is/are correct? a) 1 only b) 2 only c) Both 1 and 2 d) Neither 1 nor 2

Q7. Consider the following statements: 1. 2. 3. Which one of the following was the correct chronological order of the annexation of the British Empire? a) 1-2-3 b) 3-1-2 c) 1-3-2 d) 3-2-1

Q8. Which one of the following provisions was not involved in the treaty of Srirangapatnam between and the British? a) Tipu should give the war expenditure worth three crore rupees to the British. b) Tipu should give half of his State to the British. c) Tipu's two sons will be hostages of the British. d) Tipu should accept the Subsidiary Alliance.

Q9. Consider the following pairs 1. First Anglo-Mysore War-Treaty of Madras 2. Second Anglo-Mysore War- Treaty of Srirangpattana 3. Third Anglo-Mysore War- Treaty of Mangalore Which of the pair given above is/are Incorrect? a) 1 only b) 2 only c) 1 and 2 only d) 2 and 3 only

Q10. Which among the following events happened earliest? a) First Anglo-Sikh war b) First Anglo-Burma War

©Jatin Verma All Rights Reserved. https://www.jatinverma.org P a g e | 3

c) Second Anglo-Mysore war d) Second Anglo War

Q11. With reference to the , Consider the following statements 1. The doctrine of lapse was the pro-imperialist approach to expand the realm of the British Kingdom in India. 2. Lord Dalhousie was the originator of the Doctrine of Lapse. 3. The state of Satara was the first state annexed under the Doctrine of Lapse by Governor-General Lord Dalhousie. Which of the statement(s) given above is/are correct? a) 1 and 2 only b) 2 and 3 only c) 2 only d) 1 and 3 only

Q12. With reference to the Maratha Confederacy Consider the following statements 1. had started a confederacy of prominent Maratha chiefs to manage the rapidly expanding Maratha Power. 2. of Indore, of Baroda and Gaekwad of Nagpur were some of the prominent Maratha families emerged during this period. Which of the statement(s) given above is/are Incorrect? a) 1 only b) 2 only c) Both 1 and 2 d) Neither 1 nor 2

Q13. With reference to the Maratha Confederacy Consider the following statements 1. Peshwa Balaji Vishwanath had started a confederacy of prominent Maratha chiefs to manage the rapidly expanding Maratha Power. 2. Holkars of Indore, Bhonsle of Baroda and Gaekwad of Nagpur were some of the prominent Maratha families emerged during this period. Which of the statement(s) given above is/are Incorrect? a) 1 only b) 2 only c) Both 1 and 2 d) Neither 1 nor 2

Q14. Consider the following statements 1. In the , Ahmed Shah Abdali defeated Marathas. 2. Tipu Sultan was killed in the Fourth Anglo-Mysore War. 3. Mir Jafar entered into a conspiracy with the English for the defeat of Nawab Siraj-ud-Daulah in the . Which of the statement(s) given above is/are Incorrect? a) 1 and 3 only

©Jatin Verma All Rights Reserved. https://www.jatinverma.org P a g e | 4

b) 2 only c) 2 and 3 only d) None of the above

Q15. Who among the following Indian rulers established embassies in foreign countries on modern lines? a) Bahadur Shah II b) Haider Ali c) Mir Jafar d) Tipu Sultan

Q16. Which one of the following pairs is correctly matched? a) Battle of Porto Novo- The vs French East India Company b) Battle of - Nizam vs c) Battle of Wandiwash- East India Company vs French East India company d) Battle of Calicut- Portuguese vs French East India Company

Q17. With reference to the Anglo-Sikh War, Which one of the following provision was not included in the Treaty of ? 1. War indemnity of more than 5 crores of Rupees was to be given to the English. 2. The strength of the Sikh army was reduced. 3. The Jalandhar Doab between the Beas and the Sutlej was annexed to the Company’s dominions. 4. A Council of Regency for was set up. Select the correct code using the option given below a) 1 and 2 only b) 1 and 3 only c) 2 and 3 only d) 1 and 4 only

Q18. Which of the following pairs given below is not correct? 1. Anglo-Nepalese Relations-Treaty of Lhasa 2. Anglo-Burmese Relations- Treaty of Yandabo 3. Anglo-Tibetan Relations-Treaty of Sagauli Select the correct code using the option given below a) 1 and 2 only b) 2 and 3 only c) 1 and 3 only d) All of the above

Q19. With reference to the Sanyasi Revolt, Consider the following statement 1. Only participated in the Sanyasi Revolt. 2. This rebellion was centred in the forests of . 3. Anandamath, a semi-historical novel by Bankim Chandra Chattopadhyay, is based on the Sanyasi Revolt. Which of the statement(s) given above is/are correct?

©Jatin Verma All Rights Reserved. https://www.jatinverma.org P a g e | 5

a) 1 only b) 2 only c) 3 only d) 1 and 3 only

Q20. Which of the following statement(s) is not correct with reference to the Poligars Revolt? a) Poligars were feudal lords appointed as military chiefs and administrative governors from the time of the Vijayanagara Empire. b) The first rebellion broke out in September 1799 in district. c) Narayan Deo, the Poligar of Panjalankurichi, led the insurrection between 1795 and 1799. d) The main centres of Poligars uprising were Tirunelveli, Ramanathapuram, Sivaganga, Sivagiri, Madurai and North Arcot

Q21. With reference to the Paika Rebellion, consider the following statements 1. Paikas under the leadership of Baxi Jagabandhu rose in rebellion in March 1817. 2. The Paikas of were peasants evicted from the land. Which of the statement(s) given above is /are Incorrect? a) 1 only b) 2 only c) Both 1 and 2 d) Neither 1 nor 2

Q22. With reference to the Kukas, consider the following statements 1. The Kukas, also called Namdharis, were a sect within . 2. The concept of Swadeshi and Non-cooperation were propagated by the Kukas Which of the statement(s) given above is /are Incorrect? a) A. 1 only b) B. 2 only c) C. Both 1 and 2 d) D. Neither 1 nor 2

Q23. Correct the chronological sequence of the following events 1. Revolt of Momarias 2. Sanyasi Revolt 3. Paika Rebellion 4. Ahom Revolt Select the correct code using the option given below a) 2-1-4-3 b) 2-1-3-4 c) 1-2-3-4 d) 1-3-4-2

©Jatin Verma All Rights Reserved. https://www.jatinverma.org P a g e | 6

Q24. Consider the following events: 1. Afghan Wars 2. Barrackpore mutiny 3. Revolts in the British Indian Army in Bengal 4. mutiny Which of the following is the correct chronological sequence of the above events? a) 4-1-2-3 b) 3-4-2-1 c) 4-2-1-3 d) 3-1-2-4

Q25. With reference to the participation of the civilian population in the revolt of 1857, consider the following statements: 1. There was no significant participation of the civilian population in the north-western provinces and eastern India. 2. The peasants and small zamindars attacked the money-lenders and zamindars. Which of the statement(s) given above is/are correct? a) 1 only b) 2 only c) Both 1 and 2 d) Neither 1 nor 2

Q26. Which of the following were the results of the revolt of 1857? 1. Beginning of the era of annexations and expansion. 2. Recognition of the paramountcy of the British Crown by the Indian states. 3. Reformation of British of British military policy 4. Assumption of the by the sovereign of Great Britain. Select the correct answer using the code given below: a) 1, 3 and 4 only b) 2 and 4 only c) 2, 3 and 4 only d) 1, 2, 3 and 4

Q27. Which of the following was/were revivalist movements? 1. The 2. The Movement 3. The 4. The Deoband movement Select the correct answer using the code given below: a) 1 and 2 only b) 3 and 4 only c) 2, 3 and 4 only d) None of the above

©Jatin Verma All Rights Reserved. https://www.jatinverma.org P a g e | 7

Q28. Which of the following were economic causes of the revolt of 1857? 1. The disintegration of the old zamindari system. 2. The annexation of Indian states by the East India Company 3. The imposition of high tariff duties on Indian-made goods by the British 4. The emergence of a class of new landlords Select the correct answer using the codes given below: a) 1, 2 and 3 only b) 1 and 4 only c) 2 and 3 only d) 1, 2, 3 and 4

Q29. Which one of the following was the result of the Religious Disabilities Act, 1856? a) It aimed at banning the practice of Sati and child marriage system in India. b) It made the conversion from to Christianism legally protected. c) It restricted the Christian to set up schools in tribal areas. d) It negated the perception of traveling overseas as a sin.

Q30. In which of the locations the most ferocious battles were fought during the revolt of 1857? 1. Bundelkhand 2. Awadh 3. 4. Western Bihar 5. Select the correct answer using the codes given below: a) 1, 2, 3, 4 and 5 b) 2 only c) 4 and 5 only d) 1, 2, 3 and 4 only

Q31. With reference to the General Service Enlistment Act, which of the following statements is/are correct? 1. It was enacted during the time of Lord Canning. 2. It made all future recruits of the Bengal army to be ready for service both within and outside India. Select the correct answer using the code given below: a) 1 only b) 2 only c) Both 1 and 2 d) Neither 1 nor 2

Q32. Consider the following statement 1. The Native Marriage Act or civil Marriage Act of 1872 prohibits child marriage. 2. The Age of Consent Act of 1891 forbade the marriage of girls below the age of 12. 3. The Child Marriage Restraint Act or Sarda Act of 1929 fixed the age of marriage for girls at 14 years and for boys at 18 years.

©Jatin Verma All Rights Reserved. https://www.jatinverma.org P a g e | 8

Which of the statement(s) given above is/are Incorrect? a) 1 only b) 1 and 3 only c) 2 and 3 only d) None of the above

Q33. Consider the following pairs: Organisation Founder 1. Bharat Stree Mahamandal - Sarla Devi Chaudhurani 2. Ladies Social Conference - Saraswati 3. Arya Mahila Samaj - Ramabai Ranade Which of the pairs given above is/are correct? a) 1 only b) 1 and 2 only c) 2 and 3 only d) 3 only

Q34. Consider the following statement: 1. Jagannath Shankar Seth was active promoters of girls education in . 2. Vishnu Shastri started Satyaprakash, a weekly in Gujarati. 3. Ishwar Chandra Vidyasagar was the most prominent campaigner of Widows Remarriage Act. Which of the statement(s) given above is/are Incorrect? a) 1 only b) 2 only c) 1 and 2 only d) 2 and 3 only

Q35. With reference to the Revolt of 1857, Consider the following statement: 1. At , the symbolic leadership was to the Mughal emperor, Bahadur Shah, but the real command was led by General . 2. took over the reign of . 3. rose under , Tantia Tope, Azimullah Khan. 4. Rani Laxmibai assumed the leadership of the sepoys at . Which of the following statement(s) is correct? a) 1, 2 and 3only b) 1, 3 and 4 only c) 1 and 3 only d) 2 and 4 only

Q36. Who among the following called the Revolt of 1857 as the ‘First War of Indian Independence’? a) Aurobindo Ghosh b) c) d)

©Jatin Verma All Rights Reserved. https://www.jatinverma.org P a g e | 9

Q37. Who among the following was Governor-General during the Revolt of 1857? Who among the following was Governor-General during the Revolt of 1857? a) Lord Irwin b) Lord Lytton C c) Lord Canning d) Lord Willingdon

Q38. What was/were the objects of Queen Victoria’s Proclamation (1858)? 1. To expand the British power in India. 2. To regulate East India Company trade with India. 3. To place the Indian administration under the British Crown. Select the correct answer using the option given below a) 1 only b) 1 and 2 only c) 1 and 3 only d) All of the above

Q39. Consider the following statement 1. Sri Narayan Guru organised the Self-Respect Movement. 2. Br. Ambedkar established the Bahishkrit Hitakarni Sabha. 3. E.V. Ramaswamy Naickar in Kerala led a struggle against upper caste domination. Which of the statement(s) given above is/are Incorrect? a) 1 only b) 2 only c) 1 and 3 only

Q40. Which of the following act abolished all laws affecting the rights of persons converting to another religion or caste? a) Abolition of Sati (1829) b) Hindu Widow Remarriage Act (1856) c) Religious Disabilities Act of 1850 d) None of the above

Q41. Correct the chronological sequence of the following events 1. Establishment of Indian Women’s University 2. Formation of the National Council of Women 3. Woods Despatch on Education 4. Establishment of the Bethune School Select the correct code using the option given below a) 4-2-1-3 b) 4-3-1-2 c) 2-4-1-2 d) 1-3-2-4

©Jatin Verma All Rights Reserved. https://www.jatinverma.org P a g e | 10

Q42. Which of the following was the Impact of 1857 revolt? Which of the following was the Impact of 1857 revolt? a) Abolished the doctrine of Lapse. b) Termination of Peshwaship and the Mughal rule. c) Control of Indian administration was passed on to the British Crown d) All of the above

Q43. ‘Servants of India Society’, an association was formed to promote education, sanitation, health care and fight the social evils of and discrimination, poverty, oppression of women and domestic abuse was founded by? a) Jyotiba Phule b) Lokmanya Tilak c) d) Babasaheb Ambedkar

Q44. Which of the following was set up by Raja Rammohan Roy? 1. Brahmo Sabha 2. Tattvabodhini Sabha 3. Atmiya Sabha Select the correct answer using the code given below: a) 3 only b) 2 only c) 1 and 3 only d) 1 and 2 only

Q45. Who of the following was/were associated actively associated with the ? 1. Mahadeo Govind Ranade 2. Keshab Chandra Sen 3. Atmaram Pandurang 4. N.G. Chandavarkar Select the correct answer using the code given below: a) 1, 2 and 3 only b) 1 and 3 only c) 2 only d) 1, 2, 3 and 4

Q46. "It was like a secret society to spread liberal ideas and encourage the breakdown of caste and communal barriers. The members of this society were people of upper castes. Often, in secret meetings, these reformers would violate caste taboos on food and touch, to get rid of the hold of caste prejudice in their lives." Which one of the following has been described in the above passage? a) Brahmo Samaj b) Paramahansa Sabha

©Jatin Verma All Rights Reserved. https://www.jatinverma.org P a g e | 11

c) d) Young Bengal Movement

Q47. With reference to the Sri Paripalana (SNDP) Movement, consider the following statements: 1. It was an example of a regional movement born out of the conflict between the depressed classes and upper castes. 2. It was started in Maharashtra. Which of the statements given above is/are correct? a) 1 only b) 2 only c) Both 1 and 2 d) Neither 1 nor 2

Q48. Consider the following statements: 1. The landholders' Society was founded to safeguard the interests of the Zamindars. 2. The Indian National Association aimed at unify Indian people in a common political programme. Which of the statements given above is/are correct? a) 1 only b) 2 only c) Both 1 and 2 d) Neither 1 nor 2

Q49. With reference to the political associations formed before the , consider the following statements: 1. These associations were generally led by English-educated professionals such as lawyers. 2. All of these associations functioned in specific parts of the country and their goals were specific region- centric. Which of the statements given above is/are correct? a) 1 only b) 2 only c) Both 1 and 2 d) Neither 1 nor 2

Q50. Which of the following personalities was/were founders of the Madras Mahajan Sabha? 1. B. Subramaniya Aiyer 2. M. Viraraghavachari 3. P. Anandacharlu Select the correct answer using the code given below: a) 1, 2 and 3 b) 1 only c) 2 and 3 only d) 2 only

©Jatin Verma All Rights Reserved. https://www.jatinverma.org P a g e | 12

Q51. Which one of the following association was organized to discuss the Indian question and influence public men in to promote Indian welfare? a) The Indian League b) The East India Association c) The Indian National Association d) None of the above

Q52. ‘ Association’ formed on the demand a) To help the Ezhavas to progress materially as well as spiritually b) To preserve the status quo in socio-religious matters c) To separate representation for the lower caste in the legislature d) To train national missionaries for the service of India

Q53. Consider the following pairs Organisation Founder 1. The Poona Sarvajanik Sabha - Mahatma Jyotiba Phule 2. The Madras Mahajan Sabha - M.Viraraghavachari 3. The Indian National Association - Which of the pairs given above is/are correctly matched? a) 1 only b) 2 only c) 1 and 2 only d) 2 and 3 only

Q54. Identify the person from the description given below. 1. The great scholar of , yet accepted the western thoughts. 2. He joined the Fort William College as the head of the Sanskrit department. 3. Gave evidence from the ancient religious books for the widow remarriage. 4. Betaal Panchavinsati, Jivancharita are some of his works. Select the correct code using the option given below a) Keshav Chandra Sen b) Surendranath Banerji c) Ishwar Chandra Vidyasagar d) Justice Ranade

Q55. With reference to the social movement in India, Consider the following statement 1. A follower of Gopal Krishna Gokhale, Narayan Malhar Joshi founded the Social Service League. 2. Dev Samaj a religious and social reform society founded by Shiv Narayan Agnihotri. Which of the given above statement is/are correct? a) 1 only b) 2 only c) Both 1 and 2

©Jatin Verma All Rights Reserved. https://www.jatinverma.org P a g e | 13

d) Neither 1 nor 2

Q56. With reference to the Muslim Revivalist Movement, Consider the following statement 1. wanted to reconcile Western Scientific education with the teachings of the . 2. Sir Syed Ahmed Khan led the Aligarh Movement based on the principles of the universal religion of all humanity. 3. The Deoband Movement was organised by the orthodox section among the Muslim ulema. Which of the statement(s) given above is/are Incorrect? a) 1 only b) 2 and 3 only c) 1 and 2 only d) 1 and 3 only

Q57. Consider the following statement 1. The was an offshoot of the . 2. It aimed to oppose the suppressive policy of the British. 3. The Akali Movement was a regional movement. Which of the statement(s) given above is/are correct? a) 1 only b) 1 and 2 only c) 2 and 3 only d) 1 and 3 only

Q58. Shodak Samaj organized a) A entry movement in b) A in Gujarat c) An anti-caste movement in Maharashtra d) A movement for upliftment of tribals in Bihar

Q59. Which of the following is not correctly matched? a) Brahmo Samaj - Reformist Movements b) Prarthana Samaj - Reformist Movements c) Aligarh Movement - Reformist Movements d) Arya Samaj - Reformist Movements

Q60. Who among the following established the Tattvabodhini Sabha in 1839 at Calcutta? a) Keshab Chandra Sen b) Debendra Tagore c) Rammohan Roy d) Sivananthan Sari

Q61. Who among the following was popularly known as ‘Lokahitawadi'? a) Jyotiba Phule

©Jatin Verma All Rights Reserved. https://www.jatinverma.org P a g e | 14

b) Pt. Iswar Chandra Vidhyasagar c) Gopal Hari d)

Q62. Which of the following is associated with the Young Bengal Movement? a) Derozians b) Mitra Bandhu c) Native marriage Act d) Bethune School

Q63. The Parsi Reform Movement Rahnumai Mazdayasnan Sabha (Religious Reform Association) was founded in which year? a) 1829 b) 1851 c) 1879 d) 1861

Q64. Who among the following leaders opined that the Indian National Congress was born out of a conspiracy to abort a popular uprising in India and the bourgeois leaders were a party to it? a) b) R.P. Dutt c) Sisir Kumar Ghosh d) Dinshaw E. Wacha

Q65. Consider the following statements: 1. The Moderate leaders believed that political connections with Britain were in India's interest and they wanted to transform the colonial rule to be as close to a national rule as possible. 2. The extremist leaders like Lala Lajpat Rai was against the concept of releasing the growing discontent of the Indians through a platform like the Indian National Congress. Which of the statements given above is/are correct? a) 1 only b) 2 only c) Both 1 and 2 d) Neither 1 nor 2

Q66. Which of the following was subject matters of the campaign of the Moderates? 1. Demand for better treatment for Indian labour abroad in other British colonies. 2. Call for an increase in expenditure on elementary and technical education. 3. Criticism of the annexation of Burma by British. 4. Criticism of an expensive and time-consuming judicial system. Select the correct answer using the code given below: a) 2, 3 and 4 only

©Jatin Verma All Rights Reserved. https://www.jatinverma.org P a g e | 15

b) 1 and 2 and 4 only c) 1 and 3 only d) 1, 2, 3 and 4

Q67. Which of the following statements is correct in the context of the ? 1. The disagreements among the leaders of the Indian National Congress over course of the movement led to the Surat split. 2. The Swadeshi movement had its genesis in anti-partition protest in Bengal and thus remained restricted to Bengal only. 3. The Swadeshi movement found a wide participation of students and urban middle class but lacked the participation of women and workers. Select the correct answer using the code given below: a) 2 only b) 2 and 3 only c) 1 and 2 only d) 1 and 3 only

Q68. With reference to the Indian national movement, 'Abhinav Bharat' was: a) A movement against caste and other social evils. b) A secret society of the revolutionaries c) An anti-British newspaper published by the extremists d) Concept of as envisaged by moderates

Q69. Which of the following statements is/are correct regarding the Congress Benares session of 1905? 1. It was presided by G.K. Gokhale. 2. Self-government or Swaraj was declared as the goal of Congress at this session. 3. At the session, it was decided to extend the Swadeshi movement outside Bengal. Select the correct answer using the code given below. a) 2 only b) 1 and 3 only c) 2 and 3 only d) 1 only

Q70. The basic tenets of militant school of thought, which emerged after the failure of the peaceful and constitutional agitation of moderates, were: 1. Hatred for foreign rule 2. Individual sacrifices and not the capacity of masses to challenge the authority 3. Swaraj as the goal of the national movement Which of the statements given above is/are correct? a) 1 only b) 2 and 3 only c) 1 and 3 only d) 1, 2 and 3

©Jatin Verma All Rights Reserved. https://www.jatinverma.org P a g e | 16

Q71. Consider the following statements: 1. The Indian National Congress was established in 1885. 2. The first session of the Indian national congress was organized in 1886 and it was presided by Womesh Chandra Bonnerjee. Which of the statements given above is/are correct? a) 1 only b) 2 only c) Both 1 and 2 d) Neither 1 nor 2

Q72. Consider the following statement 1. The Committee for Indian Independenceaimed aimed to mobilise the Indian settlers abroad to send volunteers and arms to India. 2. The Ghadr party was a militant organization with headquarters at Berlin. Which of the statement(s) given above is/are correct? a) 1 only b) 2 only c) Both 1 and 2 d) Neither 1 nor 2

Q73. Consider the following pairs Organisation Founder 1. Young India Society - V.D.Savarkar 2. Anjuman I Mohisban I Watan - Ashfaqullah Khan 3. - Which of the pairs given above is/are Incorrect? a) 1 only b) 1 and 2 only c) 2 only d) 2 and 3 only

Q74. With reference to the Simla Deputation, Consider the following statement 1. Simla Deputation was a group of Muslim elites led by the . 2. This deputation demanded separate electorates for the Muslims. Which of the statement(s) given above is/are Incorrect? a) 1 only b) 2 only c) Both 1 and 2 d) Neither 1 nor 2

Q75. Which of the following statement(s) is/are correct regarding the first session of Indian National Congress? 1. It was held in and was attended by 72 delegates 2. It was presided over by W.C Banerjee and A.O.Hume assumed the charge of General Secretary.

©Jatin Verma All Rights Reserved. https://www.jatinverma.org P a g e | 17

3. Lord Randolph Churchill was the Secretary of State for India when the first session of the INC was held in 1885. Select the correct code using the code given below a) 1 and 3 only b) 2 only c) 2 and 3 only d) 1, 2 and 3

Q76. Consider the following pairs Session President 1. Calcutta Session (1906) - Gopal Krishna Gokhale 2. Surat Session (1907) - 3. Lahore Session (1909) - Madan Mohn Malviya Which of the pairs given above is/are correct? a) 1 only b) 1 and 2 only c) 2 and 3 only d) 1, 2 and 3

Q77. Correct the chronological sequence of the following events 1. Bomb thrown at Viceroy Hardinge by Rasbehari Bose and Sachin Sanyal. 2. Yugantar the revolutionary weekly started. 3. Marathi daily was started by Lokmanya Tilak. 4. Mitra Mela merged with Abhinav Bharat. Select the correct code using the option given below a) 3-4-2-1 b) 4-2-3-1 c) 3-2-4-1 d) 2-3-1-4

Q78. Consider the following statements about differences between Moderated and Extremists? 1. The social base of moderates was Zamindars and upper-middle classes in towns whereas the social base of extremists were middle and lower-middle classes in towns. 2. Moderates took ideological inspiration from European history whereas extremists took ideological inspiration from Indian History. 3. Moderates professed loyalty to the British crown whereas extremists believed that the British crown was unworthy of claiming Indian loyalty. Which of the statement(s) given above is/are correct? a) 1 and 2 only b) 1 and 3 only c) 2 and 3 only d) 1, 2 and 3

Q79. Consider the following statements about differences between Moderated and Extremists?

©Jatin Verma All Rights Reserved. https://www.jatinverma.org P a g e | 18

1. The social base of moderates was Zamindars and upper-middle classes in towns whereas the social base of extremists were middle and lower-middle classes in towns. 2. Moderates took ideological inspiration from European history whereas extremists took ideological inspiration from Indian History. 3. Moderates professed loyalty to the British crown whereas extremists believed that the British crown was unworthy of claiming Indian loyalty. Which of the statement(s) given above is/are correct? a) 1 and 2 only b) 1 and 3 only c) 2 and 3 only d) 1, 2 and 3

Q80. With reference to Bal Gangadhar Tilak, Consider the following statement: 1. Tilak was founding member of Indian National Congress. 2. The slogan ‘Swarjya is my birthright and I shall have it was given by him. 3. He advocated for bringing lower middle classes and workers into the Congress fold. Which of the statement(s) given above is/are correct? a) 1 only b) 2 only c) 2 and 3 only d) 1, 2 and 3

Q81. Which of the following pairs are correctly matched? Cases Accused 1. Attempt to the murder of Kingsford - Anant Kanhere 2. Murder of Jackson - and Praful Chaki 3. The assassination of - 4. Hardinge Bomb Case - Ras Behari Bose Select the correct code using the option given below a) 1 and 3 only b) 2 and 4 only c) 3 and 4 only d) 1, 3 and 4 only

Q82. With respect to the Indian freedom struggle, was associated with a) Ghadr Movement b) Conspiracy c) Bombay textile mills strike in 1919 d)

©Jatin Verma All Rights Reserved. https://www.jatinverma.org P a g e | 19

Q83. Which among the following events takes place during Lytton’s time? 1. Grand Delhi Durbar 2. Vernacular Press Act 3. Indian Universities Act 4. Official Secrets Act 5. Arms Act Select the correct answer using the code given below. a) 1 and 2 only b) 2, 4 and 5 only c) 1, 2 and 5 only d) 1, 3, 4 and 5 only

Q84. Consider the following statements with respect to : 1. He was the founder member of the Association. 2. He was the first Muslim who presided the annual session of congress. 3. He was associated with Muslim league. Which of statement(s) given above is/are corrected? a) 1 only b) 1 and 2 only c) 2 and 3 only d) 1, 2 and 3

Q85. With reference to the nationalist response to British participation in the First World War, consider the following statements: 1. The moderates supported the British while the extremists were against it. 2. The revolutionaries decided to utilize the opportunity to wage a war on British rule and liberate the country. Which of the statements given above is/are correct? a) 1 only b) 2 only c) Both 1 and 2 d) Neither 1 nor 2

Q86. With reference to the Home Rule League Movement, consider the following statements: 1. These leagues were organized on the lines of the Irish Home Rule Leagues. 2. They represented the emergence of a new trend of passive politics. Which of the statements given above is/are correct? a) 1 only b) 2 only c) Both 1 and 2 d) Neither 1 nor 2

Q87. Which of the following factors led to the formation of the Home Rule Movement during the Indian struggle for independence?

©Jatin Verma All Rights Reserved. https://www.jatinverma.org P a g e | 20

1. High taxation and a rise in prices 2. Montagu–Chelmsford Reforms 3. Absence of the requirement of a popular pressure among all nationalists leaders. Select the correct answer using the code given below: a) 3 only b) 1 only c) 1 and 2 only d) 2 and 3 only

Q88. Which of the following statements is/are correct about the ? 1. It allowed political activists to be tried without juries or even imprisoned without trial. 2. Gandhi termed the act as ‘the Black Act’. 3. Gandhi tried to use the young forces of the Home Rule Leagues in the Rowlatt . Select the correct answer using the code given below: a) 3 only b) 1 and 2 only c) 1 and 3 only d) 1, 2 and 3

Q89. With reference to the second session the Indian National Congress, consider the following statements: 1. At this session, extremists were accepted back into Congress. 2. It was decided to oppose the Muslim League's demand for separate electorates for Muslims. 3. It was presided by . Which of the statements given above is/are correct? a) 2 only b) 1 and 2 only c) 1 only d) 1, 2 and 3

Q90. Consider the following pairs: Newspaper founder 1. Al Hilal: - Mohammad Ali 2. Comrade: - Maulana Azad 3. Rast Goftar: - Kharshedji Cama Which of the pairs given above is/are correctly matched? a) 3 only b) 1 and 3 only c) 2 only d) 1, 2 and 3

Q91. For which of the following purpose the Hunter Committee was appointed by the British Government? a) To review the reforms made in the field of education and civil services.

©Jatin Verma All Rights Reserved. https://www.jatinverma.org P a g e | 21

b) To investigate the Punjab atrocities. c) To study constitutional reform in Britain's largest and most important possession. d) To review the reforms made in the field of education.

Q92. With reference to the Khilafat Non-Cooperation Movement, consider the following statements: 1. The were not able to bring the urban Muslims into the national movement. 2. It gave strength to the divisive forces in national politics. 3. The Chauri-Chaura incident prompted Gandhi to withdraw this movement. Which of the statements given above is/are correct? a) 1 only b) 3 only c) 2 and 3 only d) 1 and 3 only

Q93. With reference to the moderates, consider the following statement 1. They believed that political connections with the British are in India’s social, political and cultural interests. 2. They used constitutional and extra-constitutional methods to achieve their objectives. Which of the statements given above is/are correct? a) 1 only b) 2 only c) Both 1 and 2 d) Neither 1 nor 2

Q94. Which among the following organizations was/were founded by ? 1. Dakshina Bharat Prachar Sabha 2. All India Anti Untouchability League 3. Natal Indian Congress Select the correct code using the option given below a) 2 only b) 1 and 3 only c) 2 and 3 only d) 1, 2 and 3

Q95. The object of the Hunter Commission was to a) Extend the provision of separate electorates, initially given only to the Muslims. b) Define the powers of the Secretary of State for India. c) To investigate the Jalianwala Bagh Massacre. d) Impose censorship on national press.

Q96. Which of the following is/are provisions of the Rowlett Act? 1. Imprisonment of Indians without trials. 2. No right to appeal for the accused against the order of the court.

©Jatin Verma All Rights Reserved. https://www.jatinverma.org P a g e | 22

3. The court of Law shall consist of European Judge exclusively. Select the correct code using the option given below a) 1 only b) 2 and 3 only c) 1 and 2 only d) 1, 2 and 3

Q97. “Marked by the surrender of Government titles, the boycott of government schools and colleges, law courts etc, it was the first open extra-constitutional programme of mass mobilization started by the Congress. The programme condemned untouchability and decided to maintain non-violence” Which of the following movement is being described in the following passages? a) Rowlett Satyagraha b) Non-Cooperation Movement c) Civil Disobedience Movement d)

Q98. In the context of Indian History, the ‘Dyarchy’ refers to a) Division of the central legislature into two houses. b) Introduction of double government i.e. Central and State governments c) Having two sets of rulers; one in and another in Delhi. d) Division of the subjects delegated to the provision into two categories.

Q99. Which of the following statement is correct regarding the “Tolstoy Farm”? a) It is a book written by Gandhi about his experiences in . b) It is a place where Gandhi was kept under house arrest. c) It was set up by Gandhi to house the families of the Satyagraha in South Africa. d) It was a farm where Indians were brought as indentured labourers to work.

Q100. Which among the following events happened earliest? a) Case. b) Muslim League passed a resolution for demanding . c) Outbreak of the First World War. d) Chittagong armoury raid.

Q101. In which session of the Indian National Congress did the historic union of Congress and Muslim take place? a) Tripuri Session, 1939 b) Lahore Session, 1940 c) Session, 1946 d) Lucknow Session, 1916

Q102. With reference to the Nationalist Response to British participation in the First World War, Consider the following statement: 1. While the moderates supported Britain in the war, extremist opposed the move.

©Jatin Verma All Rights Reserved. https://www.jatinverma.org P a g e | 23

2. The revolutionaries saw the war as an opportunity to wage a fight against British rule and liberate the country. Which of the statement given above is/are correct? a) 1 only b) 2 only c) Both 1 and 2

Q103. In response to which of the following incidents did Mahatma Gandhi withdrew the Non- Cooperation Movement? a) Jalianwala Bagh Committee b) Chauri Chaura Outrage c) Announcement of Morley-Minto Reforms d) The signing of Gandhi-Irwin Pact

Q104. Mahatma Gandhi organized a satyagraha on behalf of the peasants of Champaran to: 1. Oppose exploitation by the European planters under the Tinkkathia System. 2. Conduct a detailed inquiry into the condition of the cotton sharecroppers in that region. Select the correct code using the option given below a) 1 only b) 2 only c) Both 1 and 2 d) Neither 1 nor2

Q105. Who of the following was/were in the group called 'Swarajists'? 1. M.A. Ansari 2. 3. C.R. Das 4. Ajmal Khan Select the correct answer using the code given below: a) 1 and 2 only b) 2 and 3 only c) 2, 3 and 4 only d) 1, 2, 3 and 4

Q106. Consider the following statements: 1. Gandhi did not oppose to the Swarajist proposal of council entry. 2. The Swarajists were not allowed to contest elections as a group within the Congress so they formed a separate political party. 3. The Swarajists aimed at presenting the nationalist demand of self-government in councils. Which of the statements given above is/are correct? a) 2 only b) 1 and 2 only c) 3 only d) 2 and 3 only

©Jatin Verma All Rights Reserved. https://www.jatinverma.org P a g e | 24

Q107. Why did Gandhi agree to the Swarajist proposal of council entry? 1. Gandhi felt public opposition to the programme of council entry would be counter-productive. 2. The swarajists had managed to win all 141 seats to the central legislative council. 3. There was a government crackdown on the Swarajists. Select the correct answer using the code given below: a) 1 and 3 only b) 2 only c) 1 and 2 only d) 1, 2 and 3

Q108. During the 1920s there was a rise of the left-wing groups, with reference to those groups, consider the following statements: 1. They were supporters of No-Changers. 2. They stressed in purna swarajya. 3. They stressed the need to combine nationalism and anti-imperialism with social justice. Which of the statements given above is/are correct? a) 1 only b) 3 only c) 1 and 2 only d) 2 and 3 only

Q109. Which among the following were founders of the Hindustan Republican Association? 1. Jogesh Chandra Chatterjee 2. 3. Ramprasad Bismil 4. Sachin Sanyal Select the correct answer using the code given below: a) 1, 3 and 4 only b) 2 and 3 only c) 2 and 4 only

Q110. Who among the following was the writer of the book 'The Philosophy of the Bomb'? a) Bhagwaticharan Vohra b) Sukhdev c) Bhagat Singh d) Lala Lajpat Rai

Q111. Which of the following was/were recommendations of the ''? 1. Complete self-rule independent of the British Empire. 2. A parliamentary system of government. 3. An independent judiciary with a supreme court at its head. Select the correct answer using the codes given below:

©Jatin Verma All Rights Reserved. https://www.jatinverma.org P a g e | 25

a) 2 only b) 1 and 3 only c) 2 and 3 only d) 1, 2 and 3

Q112. For which of the following reason/reasons the session of the Indian National Congress was important? 1. The Indian National Congress leaders resolved to boycott the at this session. 2. The Delhi Pact was endorsed at this session. 3. The Indian National Congress dissociated itself from political violence. Select the correct answer using the code given below: a) 2 and 3 only b) 1, 2 and 3 c) 1 and 2 only d) None of the above

Q113. Correct the chronological sequence of the following events 1. Second Round Table Conference 2. The Delhi Pact 3. Irwin’s Declaration 4. Dandi March Select the correct code using the option given below a) 4-2-3-1 b) 3-4-2-1 c) 3-2-4-1 d) 2-4-3-1

Q114. Consider the following statements regarding the Swarajists 1. Leaders advocating entry into legislative councils came to be known as the Swarajists. 2. Swarajists school of thought was led by , , C.Rajagopalachari and M.A.Ansari. Which of the statement given above is/are correct? a) 1 only b) 2 only c) Both 1 and 2 d) Neither 1 nor 2

Q115. With reference to the Indian National Congress annual session, Consider the following pairs 1. Kanpur Session (1925)- Resolution passed 2. Lahore Session (1929)- First Indian woman president of Indian National Congress 3. Karachi Session (1931)- Resolution on National Economic Programme passed Which of the given above pairs is/are not correctly matched? a) 1 and 2 only b) 2 and 3 only

©Jatin Verma All Rights Reserved. https://www.jatinverma.org P a g e | 26

c) 3 only d) None of the above

Q116. Who among the following attended all the three Round Table Conferences held in London? 1. Mahatma Gandhi 2. Dr B.R.Ambedkar 3. Tez Bahadur Sapru 4. Select the correct using the option given below a) 1 and 2 only b) 2 and 3 only c) 3 and 4 only d) All of the above

Q117. With reference to the Swarajists within the legislature, consider the following statements 1. Vittalbhai Patel was elected speaker of central Legislative Assembly. 2. They outvoted the Government several times, even on matters relating to budgetary grants, and passed adjournment motions. 3. A noteworthy achievement was the defeat of the Public Safety Bill. Which of the statement given above is/are correct? a) Only 1 b) Only 2 c) Only 3 d) All of the above

Q118. with reference to the no-changers school of thought, Consider the following statements 1. Opposed council entry. 2. Concentration on constructive work. 3. Continuation of the philosophy of boycott and noncooperation. 4. C.R Das and Motilal Nehru were their main leaders. Which of the statement given above is/are correct? a) 1, 2 and 3 b) 2, 3 and 4 c) 1, 3 and 4 d) All of the above

Q119. Who led a from Trichinopoly to Vedaranniyam on the Tanjore coast in ? a) Surya Sen b) K Kelappan c) P Krishna Pillai d) C. Rajagopalachari

Q120. Which of the following was/were members of the Nehru Committee report? 1. Vallabhbhai Patel

©Jatin Verma All Rights Reserved. https://www.jatinverma.org P a g e | 27

2. Subhash Chandra Bose 3. Tej Bahadur Sapru Select the correct code using the option given below a) 1 only b) 1 and 2 only c) 2 and 3 only d) 1, 2 and 3

Q121. Which of the following was/were the demands put forward by Mahatma Gandhi during the Civil Disobedience Movement? 1. Prohibition of intoxicants and liquor 2. Introduce textile protection 3. Abolish salt tax Select the correct code using the option given below a) 1 and 2 only b) 1 and 3 only c) 3 only d) 1, 2 and 3

Q122. Consider the following statement 1. Unlike the Non-cooperation Movement, complete independence was not the objective of the Civil Disobedience Movement 2. Unlike the Non-cooperation Movement, there was large scale Muslim participation. Which of the statement given above is/are correct? a) 1 only b) 2 only c) Both 1 and 2 d) Neither 1 nor 2

Q123. With reference to the nationalist response to the British participation in the First World War, consider the following statement 1. While the moderates supported Britain in the war, extremists opposed the move. 2. The revolutionaries saw the war as an opportunity to wage a fight against British rule and liberate the country. Which of the statement given above is/are correct? a) 1 only b) 2 only c) Both 1 and 2 d) Neither 1 nor 2

Q124. With reference to the left-wing of the Indian freedom movement, consider the following pairs Leader Publication 1. Muzzafar Ahmed - Inquilab 2. S.A.Dange - The Socialist

©Jatin Verma All Rights Reserved. https://www.jatinverma.org P a g e | 28

3. M. Singaravelu - Labour Kisan Gazette Which of the pairs given above is/are correctly matched? a) 1 only b) 2 Only c) 2 and 3 only d) 1, 2 and 3

Q125. Why was the Civil Disobedience movement after the second Round Table Conference short-lived? 1. Only Gandhi was the leader who actively built up the tempo of the movement. 2. The masses were not prepared. 3. The people had lost political faith in the Indian National Congress. Select the correct answer using the code given below: a) 3 only b) 2 only c) 1 and 3 only d) 1 and 2 only

Q126. Who of the following personality/personalities was/were associated with Poona Pact? 1. 2. B. R. Ambedkar 3. Chakravarti Rajagopalachari Select the correct answer using the codes given below. a) 2 and 3 only b) 1 and 2 only c) 2 only d) 1, 2 and 3

Q127. Which of the following was/were in the Reserved subjects mentioned in the Government of India Act, 1935? 1. Tribal areas 2. Defence 3. Foreign affairs Select the correct answer using the code given below: a) 3 only b) 1 and 2 only c) 2 and 3 only d) 1, 2 and 3

Q128. Consider the following statements: 1. Subhash Chandra Bose resigned from Congress after his defeat in the election for the Presidentship of Tripuri session. 2. The Forward Block was a new party outside the Indian National Congress. Select the correct answer using the code given below:

©Jatin Verma All Rights Reserved. https://www.jatinverma.org P a g e | 29

a) 1 only b) 2 only c) Both 1 and 2 d) Neither 1 nor 2

Q129. Why did Gandhi not initiate a movement immediately after the beginning of World War-II? a) Gandhi felt that the masses were not ready for the struggle at that moment. b) Gandhi believed that there was an absence of Hindu-Muslim unity in the country. c) Gandhi felt that the August offer is just and will be fulfilled by the British government. d) Gandhi and some other dominant leaders of the Indian National Congress felt that the cause of Allies was just.

Q130. Which of the following statement is incorrect about the August offer? a) It accepted INC's demand for setting up a national government at the Centre. b) It proposed to appoint a representative body to prepare the Constitution after the World war. c) It assured to increase the number of Indians in the Viceroy's Executive council. d) It assured to set up a war advisory Council.

Q131. Consider the following statements: The Individual Satyagraha was launched 1. To give another opportunity to the Government to accept Congress' demands peacefully. 2. To show that nationalist patience was not due to weakness. Which of the statements given above is/are correct? a) 1 only b) 2 only c) Both 1 and 2 d) Neither 1 nor 2

Q132. With reference to the of 1942, consider the following statements: 1. It mentioned about the creation of Pakistan. 2. It proposed a single Indian Union. 3. It offered a Dominion status to India. Select the correct answer using the code given below: a) 2 and 3 only b) 1 only c) 1 and 2 only d) 1, 2 and 3

Q133. With reference to the Government of India Act, 1935, Consider the following statement 1. It was to comprise all British Indian provinces, all chief commissioner’s provinces and the princely states.

©Jatin Verma All Rights Reserved. https://www.jatinverma.org P a g e | 30

2. Executive councillors were responsible to the Central Legislature. Which of the statement(S) given above is/are Incorrect? a) 1 only b) 2 only c) Both 1 and 2 d) Neither 1 nor 2

Q134. Which of the following events take place under the reign of Viceroys Lord Willingdon? 1. Third Round Table Conference. 2. Establishment of 3. Lahore session of the congress 4. Enactment of the Government of India Act 1935 Select the correct code using the option given below a) 1 and 2 only b) 2 and 3 only c) 1, 2 and 3 only d) 1, 2 and 4 only

Q135. With reference to the Indian Freedom Struggle, Consider the following statement 1. The British Government came up with August Offer to get the cooperation of India in the War effort. 2. Cripps Mission refused dominion status to India. Which of the following statement given above is/are Incorrect? a) 1 only b) 2 only c) Both 1 and 2 d) Neither 1 nor 2

Q136. Correct the chronological sequence of the following events 1. Haripura INC Session 2. August Offer 3. Second World War 4. Cripps Mission Select the correct code using the option given below a) 1-2-4-3 b) 2-1-3-4 c) 1-3-2-4 d) 1-3-4-2

Q137. With reference to the Individual satyagraha, consider the following statement 1. The aims of launching individual satyagraha were to express people's feeling that they were not interested in the war. 2. was the first to offer the satyagraha. Which of the statement(s) given above is/are correct? a) 1 only

©Jatin Verma All Rights Reserved. https://www.jatinverma.org P a g e | 31

b) 2 only c) Both 1 and 2 d) Neither 1 nor 2

Q138. Consider the following pairs 1. Bombay Session(1934) - Rajendra Prasad 2. Faizpur Session(1936) - Subash Chandra Bose 3. Tripuri Session(1939) - Jawaharlal Nehru Which of the following pairs is/are not correctly matched? a) 1 only b) 1 and 2 only c) 2 and 3 only d) 1, 2 and 3

Q139. In which of the following provinces was a congress ministry not formed under the Government of India Act, 1935? a) Bihar b) Madras c) Orissa d) Punjab

Q140. The Cabinet mission rejected the idea of creating a separate Pakistan, because 1. Proposed Pakistan had many Non-Muslim populations. 2. The Partition would result in economic and administrative problems. 3. The regional ties of Bengal and Punjab would be disturbed. 4. The idea might lead to further balkanization of the territory of India Which of the statements given above is/are correct? a) 1 only b) 2, 3 and 4 only c) 2 and 3 only d) 1, 2, 3 and 4

Q141. Which of the following statements is/are correct about the 'Quit India' Resolution? 1. It was endorsed at the Congress annual session at Wardha on August 8, 1942. 2. It sanctioned a civil disobedience movement against British rule. Which of the statements given above is/are correct? a) 1 only b) 2 only c) Both 1 and 2 d) Neither 1 nor 2

Q142. Regarding Gandhi's general instructions to different sections of society during the Quit India movement, consider the following statements: 1. He asked government servants to resign and declare their allegiance to the Congress.

©Jatin Verma All Rights Reserved. https://www.jatinverma.org P a g e | 32

2. He asked the soldiers to leave the Army and fight for a free India. 3. He requested the princes of princely states to support the masses and accept the sovereignty of their people. Which of the statements given above is/are correct? a) 2 only b) 1 and 3 only c) 3 only d) 1, 2 and 3

Q143. With reference to the participation of students in the Quit India movement, consider the following statements: 1. They wrote and distributed illegal news sheets. 2. They responded by going on strike in schools and colleges. 3. They participated in processions. 4. They acted as couriers for underground networks. Which of the statements given above is/are correct? a) 3 only b) 1 and 4 only c) 2 and 3 only d) 1, 2, 3 and 4

Q144. Consider the following statements: 1. The Communists did not join the Quit India movement. 2. The lower levels in police and administration participated in the Quit India movement. 3. The peasants of all strata were at the heart of the movement while no zamindars participated in the quit India movement. Which of the statements given above is/are correct? a) 3 only b) 1 and 2 only c) 1 and 3 only d) 2 and 3 only

Q145. Which of the following organizations/groups opposed the quit India movement? 1. The Muslim League 2. The 3. The Princely states Select the correct answer using the code given below: a) 3 only b) 1 and 2 only c) 1 only d) 2 and 3 only

Q146. Consider the following personalities:

©Jatin Verma All Rights Reserved. https://www.jatinverma.org P a g e | 33

1. Shah Nawaz Khan 2. Prem Kumar Sehgal 3. Gurbaksh Singh Dhillon. 4. Rashbehari Bose Who of the above were associated with the ? a) 3 and 4 only b) 1 and 2 only c) 1, 2 and 3 only

Q147. Which of the following is/are correct about the Quit India Movement? 1. It was an immediate reaction of the Indian National Congress on the failure of the Cripps mission. 2. was started underground radio in Bombay during this movement. 3. No communal conflicts were observed during the period of this movement. Select the correct answer using the code given below: a) 2 only b) 1 and 2 only c) 2 and 3 only d) 1, 2 and 3

Q148. Which of the following was/were the Fundamental Cause of the 1943 Famine? 1. Non-intervention of the British government to import rice from abroad. 2. Gross mismangement and deliberate profiteering. 3. Need to feed a vast army diverted foodstuffs. Select the correct code using the option given below a) 1 only b) 1 and 2 only c) 2 and 3 only d) 1, 2 and 3

Q149. Which of the following points was/were in the C.Rajagopalachari Formula? 1. Caste Hindus and Muslims were to have equal representation. 2. Muslim League to endorse congress demand for independence. 3. League to cooperate with Congress in forming a provisional government at the centre. 4. The terms to be operative only if England transferred full powers to India. Select the correct using the option given below a) 1 and 3 only b) 1, 2 and 4 only c) 2, 3 and 4 only d) All of the above

Q150. With reference to the Quit India Resolution, Consider the following statement 1. The Quit India Resolution adopted in 1942 Wardha Congress Working Committee. 2 The Quit India Resolution proposed by Motilal Nehru. Which of the given statement given above is/are Incorrect?

©Jatin Verma All Rights Reserved. https://www.jatinverma.org P a g e | 34

a) 1 only b) 2 only c) Both 1 and 2 d) Neither 1 nor 2

Q151. Correct the chronological sequence of the following events 1. Wavell Plan 2. Rajgopalchari Formula 3. Quit India Resolution 4. Desai-Liaquat Pact Select the correct code using the option given below a) 3-4-2-1 b) 3-2-4-1 c) 2-4-3-1 d) 4-3-1-2

Q152. With reference to the Parallel Government established during the Indian Freedom Struggle, Consider the following statement 1. Parallel government was established under Chittu Pandey in Balia, the town of United Province. 2. Jatiya Sarkar was an independent parallel government established in Tamluk, Bengal. 3. Leaders like Yashwant Chavan, Nana Patil established Prati Sarkar in Nagpur. Which of the statement given above is/are correct? a) 1 only b) 1 and 2 only c) 2 and 3 only d) 1, 2 and 3

Q153. Quit India Movement launched in response to a) Cabinet Mission Plan b) Simon Commission Report c) Cripps Proposals d) Wavell Plan

Q154. Who among the following participated during the Quit India Movement? 1. Bhulabhai Desai 2. Usha Mehta 3. Subash Chandra Bose 4. Satish Samanta Select the correct code using the option given below a) 1, 2 and 3 only b) 1, 3 and 4 only c) 2, 3 and 4 only d) All of the above

©Jatin Verma All Rights Reserved. https://www.jatinverma.org P a g e | 35

Q155. With reference to the Indian Freedom Struggle, Usha Mehta is well-known for a) Running the secret congress radio in the wake of quit India movement b) Participating in the Second Round Table Conference c) Leading a contingent of Indian National Army d) Assisting in the formation of Interim Government under Pandit Jawaharlal Nehru

Q156. Which of the following not correct about Quit India Movement? a) It was a non-violent movement b) It was led by Mahatma Gandhi c) It was a spontaneous movement d) It did not attract the labour class in general

Q157. Which of the following parties were established by Dr B.R. Ambedkar? 1. The Peasants and Workers Party of India 2. All India Scheduled Castes Federation 3. The Independent Labour Party Select the correct code using the option given below a) 1 and 2 only b) 2 and 3 only c) 1 and 3 only d) 1, 2 and 3

Q158. The Congress ministers resigned in the seven provinces in 1939, because a) Congress could not form ministeries in the other four provinces b) The emergence of a ‘left-wing’ in Congress made the working of the ministries impossible c) There were widespread communal disturbances in their provinces d) None of the statements(a), (b) and (c) given above is correct

Q159. Who among the following were official Congress negotiations with Cripps Mission? a) Mahatma Gandhi and Sardar Patel b) J.B. Kripalani and C. Rajagopalachari c) Pandit Nehru nad Maulana Azad d) Dr.Rajendra Prasad and Rafi Ahmed Kidwai

Q160. After the second world war, there was a change in the British Government's Attitude, because 1. The United Kingdom emerged as a superpower. 2. There was a wave of socialist radical governments throughout Europe. 3. There was an anti-imperialist wave in South-East Asia. 4. The British soldiers were weary and tired. Which of the statements given above is/are correct? a) 1 and 4 only b) 2, 3 and 4 only c) 1 and 3 only d) 1, 2, 3 and 4

©Jatin Verma All Rights Reserved. https://www.jatinverma.org P a g e | 36

Q161. Although the Dominion Status was against the Lahore Congress (1929) spirit, the Indian National Congress accepted it, because 1. It would ensure a peaceful and quick transfer of power. 2. It would allow for some much-needed continuity in the bureaucracy and the army. 3. The Indian National Congress did not want to assume authority quickly and wanted a slow transition of power. Which of the statements given above is/are correct? a) 3 only b) 2 only c) 1 and 3 only d) 1 and 2 only

Q162. With reference to the Indian Independence Act of 1947, consider the following statements: 1. Each dominion was to have a Prime Minister to be responsible for the effective operation of the Act. 2. The existing Central Legislative Assembly and the Council of States were to exercise the powers of the legislature of that dominion. Which of the statements given above is/are correct? a) 1 only b) 2 only c) Both 1 and 2 d) Neither 1 nor 2

Q163. In which one of the following acts, the East India Company's territories in India were termed 'British possessions'? a) The Act for Better Government of India, 1858 b) The Pitt’s India Act of 1784 c) The Charter Act of 1793 d) The Regulating Act of 1773

Q164. With reference to the Charter Act of 1813, consider the following statements: 1. This act ended the East India Company’s monopoly over trade in Asia. 2. The process of giving a dividend to shareholders of the East India Company was stopped through this act. 3. The powers of the Board of Control were reduced drastically. Which of the statements given above is/are correct? a) 1 only b) 2 and 3 only c) 1 and 2 only d) None of the above

Q165. Consider the following statements: Under the Indian Councils Act, 1909 1. The strength of the Imperial Legislative Council was increased.

©Jatin Verma All Rights Reserved. https://www.jatinverma.org P a g e | 37

2. With regard to the central government, an Indian member was taken for the first time in the Executive Council of the Governor-General. 3. The income qualification for Muslim voters was kept lower than that for Hindus. Which of the statements given above is/are correct? a) 1, 2 and 3 b) 2 and 3 only c) 1 and 2 only d) 1 only

Q166. Which of the following statement is/are correct about 3rd June Plan? 1. This plan was based on the formula to divide India but retain maximum unity. 2. This plan favoured the transfer of NWFP and Sylhet district of Bengal to Pakistan. 3. This plan ruled out that successor governments would be given dominion status. Select the correct answer using the code given below: a) 1 and 2 only b) 1 and 3 only c) 2 and 3 only d) 1, 2 and 3

Q167. Which one of the following acts lifted all restrictions on European immigration and the acquisition of property in India? a) The Charter Act of 1833 b) The Charter Act of 1853 c) The Act for Better Government of India, 1858 d) The Charter Act of 1813

Q168. With reference to the Regulating Act of 1773, Consider the following statement 1. Regulate the functioning of the East India company. 2. Establishment of Supreme Court of judicature in Bengal. 3. Governor-General of Bengal exercises some powers over Bombay and Madras. Which of the following statement is/are correct? a) 1 only b) 1 and 2 only c) 2 and 3 only d) 1, 2 and 3

Q169. Which one of the following pairs is not correctly matched? a) Pitt’s India Act: b) The Doctrine of Lapse: Dalhousie c) Vernacular Press Act: Curzon d) Ilbert Bill: Ripon

Q170. Which of the following is/are the principal features(s) of the Government of India Act, 1919?

©Jatin Verma All Rights Reserved. https://www.jatinverma.org P a g e | 38

1. Introduction of diarchy in the executive government of the provinces. 2. Introduction of separate electorates for Muslims. 3. Devolution of legislative authority by the centre to the provinces. Select the correct answer using the codes given below a) 1 only b) 2 and 3 only c) 1 and 3 only d) All of the above

Q171. Who among the following Governor Generals created the Covenanted Civil Service of India which later came to be known as the Indian Civil Service? a) Warren Hastings b) Richard Wellesley c) Charles Cornwallis d) William Bentinck

Q172. Correct the chronological sequence of the Governor-General in India 1. Lord Dalhousie 2. Lord Ripon 3. Lord Dufferin 4. Lord Mayo 5. Lord Irwin Select the correct code using the option given below a) 1-4-2-3-5 b) 1-3-2-4-5 c) 1-2-4-3-5 d) 1-4-3-2-5

Q173. Consider the following statements: 1. Warren Hastings was the first Governor-General who established a regular police force in India on the British Pattern. 2. A Supreme Court was established at Calcutta by the Regulating Act, 1773. 3. The Indian Penal Code came into effect in the year 1860. Which of the statements given above is/are correct? a) 1 and 2 Only b) 2 and 3 Only c) 1 and 3 Only d) All of the above

Q174. Consider the following pairs Governor-General in India Events 1. Lord Hardinge II - Delhi Durbar

©Jatin Verma All Rights Reserved. https://www.jatinverma.org P a g e | 39

2. Lord Irwin - Visit of Simon commission 3. Lord Willingdon - Fast unto death by Gandhi in Yerwada jail Which of the pair given above is/are Incorrect? a) 1 only b) 1 and 3 only c) 3 only d) None of the above

Q175. With reference to Orientalist-Anglicist debate, consider the following statements: 1. The Anglicists were in favor of modern studies. 2. The Orientalists expansion of traditional Indian learning. 3. The Orientalists demanded for the use of vernacular languages for education. Which of the statements given above is/are correct? a) 2 only b) 1 and 3 only c) 2 and 3 only d) 1, 2 and 3

Q176. Who among the following developed a comprehensive scheme of village education through the medium of vernacular languages? a) James Thomson b) Charles wood c) Lord Macaulay d) Harcourt Butler

Q177. With reference to Press Act of 1835, consider the following statements: 1. It required the printer/publisher to give a precise account of premises of a publication to the district administration. 2. It led to the rapid growth of newspapers in India. Which of the statements given above is/are correct? a) 1 only b) 2 only c) Both 1 and 2 d) Neither 1 nor 2

Q178. Which of the following statement is incorrect about the Hunter of education Commission of 1882- 83? a) It was appointed to review the progress of education in British India. b) It's recommendations led to the controversy of Cunningham circular. c) It recommended the use of vernacular language for primary education. d) Its recommendation was mostly focussed on primary and secondary education.

Q179. With reference to Hartog Committee on education, consider the following statements:

©Jatin Verma All Rights Reserved. https://www.jatinverma.org P a g e | 40

1. It recommended the compulsory elementary education from 6 to 14 years of age. 2. It was appointed to suggest measure from preventing deterioration of education standards. Which of the statements given above is/are correct? a) 1 only b) 2 only c) Both 1 and 2 d) Neither 1 nor 2

Q180. In the history of the development of Indian press, who among the following had earned the epithet of, "liberator of the Indian press"? a) Lord Ripon b) Lord Metcalf c) Lord Wellesley d) Lord Curzon

Q181. Consider the following pairs: Institution Founder 1. Fort William College - Charles Cornwallis 2. Sanskrit college - Jonathan Duncan 3. Bethune School - John Elliot Drinkwater 4. Calcutta Madarsa - Warren Hasting Which of the pairs given above is/are correctly matched? a) 1 and 2 only b) 2, 3 and 4 c) 1 and 3 only d) 2 and 4 only

Q182. was associated with which of the following newspapers? 1. Mir’at’l-Akhbar 2. Bengal Herald 3. Sambad Kaumudi Select the correct answer using the code given below: a) 3 only b) 2 and 3 only c) 1 only d) 1, 2 and 3

Q183. Consider the following pairs: 1. Calcutta Madrasah – Warren Hastings 2. Fort William College – Arthur Wellesley 3. Sanskrit College – Jonathan Duncan Which of the following pairs given above is/are correct? a) 1 only b) 2 and 3 only

©Jatin Verma All Rights Reserved. https://www.jatinverma.org P a g e | 41

c) 3 only d) 1 and 3 only

Q184. With reference to the Woods Despatch, Consider the following statements 1. Focus on female and vocational education 2. Improve standards of University Education 3. Introduced Secular education in government institutions. Which of the statement(s) given above is/are Incorrect? a) 1 only b) 2 only c) 1 and 3 only d) 2 and 3 only

Q185. Correct the chronological sequence of the following events related to the development of Education in India 1. Hunter Education Commission 2. Saddler University Commission 3. Indian Universities Act 4. Hartog Committee Select the correct code using the option given below a) 1-3-2-4 b) 2-4-3-1 c) 1-4-3-2 d) 1-3-4-2

Q186. Objective of Rayleigh Commission was a) Define the jurisdiction of the Central and Provincial Governments. b) To impose censorship on the national press. c) To inquire into the condition and prospects of universities in India. d) Define the powers of the Secretary of State for India.

Q187. With reference to the Vernacular Press Act, 1878, Consider the following statement 1. Publisher of any vernacular newspaper to enter into a bond with government undertaking. 2. Provision of appeal in a court of law against the action of magistrate. 3. Action was taken against Som Prakash, Bharat Mihir, Dacca Praksh and Samachar under this act. Which of the statement given above is/are Incorrect? a) 1 and 2 only b) 2 only c) 2 and 3 only d) 1, 2 and 3

Q188. The Bengal Gazette or Calcutta General Advertiser, the first newspaper in India started by a) James Augustus Hickey b) Henry Vivian Derozio

©Jatin Verma All Rights Reserved. https://www.jatinverma.org P a g e | 42

c) J.S. Buckingham d) R.Williams

Q189. Consider the following pairs Name of the Paper/Journal Name of the Founder/Editor 1. Somprakasha Dwarkanath Vidyabhushan 2. Rast Goftar Dadabhai Naoroji 3. Sambad Kaumudi Devendranath Tagore Which of the pairs given above is/are correct? a) 1 only b) 2 only c) 1 and 2 only d) 3 only

Q190. With reference to the Wardha Scheme of Basic Education, consider the following statement 1. Inclusion of a basic handicraft in the syllabus. 2. Teaching to be in the vernacular language from class II to X. 3. Maulana formulated a detailed national scheme for basic education. Which of the following statement is/are correct? a) 1 only b) 1 and 2 only c) 2 and 3 only d) 1, 2 and 3

Q191. Which among the following Mughal Emperor organised the paintings in Imperial Establishment or Karkhanas? a) b) c) d) Shahajan

Q192.Which of the following monument constructed in the complex? 1. Buland Darwaza 2. Panch Mahal 3. Anup Talao 4. Moti Masjid 5. Ibadat Khana Select the correct code using the option given below a) 1, 2 and 4 only b) 2, 4 and 5 only c) 3, 4 and 5 only d) 1, 2, 3 and 5 only

Q193. Which of the following was a famous painter at Akbar’s court?

©Jatin Verma All Rights Reserved. https://www.jatinverma.org P a g e | 43

1. Mansur 2. Daswanth 3. Mukand 4. Bishandas 5. Basawan Select the correct code using the option given below a) 1, 2 and 5 only b) 1, 3 and 5 only c) 2, 3 and 5 only d) 2, 4 and 5 only

Q194. Ayar-i-Danish and Anwar-i-sunavli book written during the reign of Jahangir deal with: a) Book on General b) Book on the history of the Afghan rulers c) Book on animal fable d) Book on a sharia-based compilation on statecraft

Q195. With reference to the Cultural History of India, consider the following statement 1. Only Red sandstone was used in making the tomb of Itmad ud Daulah. 2. Red sandstone and white marble were used in making of Bibi ka Makbara. Which of the statement given above is/are correct? a) 1 only b) 2 only c) Both 1 and 2 d) Neither 1 nor 2

Q196. Identify the person from the description given below : 1. He preferred the paintings of flora and fauna, i.e. birds, animals, trees and flowers. 2. He emphasised on bringing a naturalism to portrait painting. 3. An animal fable called Ayar-i-Danish was illustrated during his reign. 4. Mansur was the famous painters in his court. Select the correct code using the option given below a) Shahjahan b) Akbar c) Humayun d) Jahangir

Q197. With reference to the art and archaeological history of India, which one among the following was made earliest? a) Tomb of Humayun b) Complex c) Fatehpur Sikri Complex d)

©Jatin Verma All Rights Reserved. https://www.jatinverma.org P a g e | 44

Q198. With reference to the Administration during the reign of King , which of the following pairs given below is not correct? a) Peshwa- Prime Minister b) Majumdar - Accountant c) Waqianavis - deals with intelligence, house affairs etc d) Panditrao - Foreign minister

Q199. With reference to the Deccan Sultanate, consider the following pairs 1. Imad Shahi Dynasty - Berar 2. Qutub Shahi Dynasty - Ahmednagar 3. - Bijapur 4. Nizam shahi - Golconda Which of the pairs given above is/are Incorrect? a) 1 and 3 only b) 1 and 2 only c) 2 and 3 only d) 2 and 4 only

Q200. Which among the following Mughal Emperor allowed English East India Company to establish its first factory in India? a) b) Akbar c) Shahjahan d) Jahangir

Q201. Which of the following is/are features of the ? 1. Charbagh Style 2. Bulbous Dome 3. Massive vaulted gateways 4. Use of Red Sandstone Select the correct code using the option given below a) 1, 2 and 3 only b) 1, 2 and 4 only c) 2, 3 and 4 only d) All of the above

Q202. Match List-I with List-II List I List II 1. Iqta 1. Marathas 2. Jagir 2. Mughals 3. Amaram 3. Delhi Sultans 4. Mokasa 4. Vijayanagara Select the correct code using the option given below a) A-3, B-2, C-1, D-4

©Jatin Verma All Rights Reserved. https://www.jatinverma.org P a g e | 45

b) A-2, B-1, C-3, D-4 c) A-3, B-2, C-4, D-1 d) A-2, B-3, C-1, D-4

Q203. With reference to the Deccan policy of Mughals, consider the following statements: 1. Aurangazeb extended his support to the Muslim Sultanates of Deccan against the Marathas. 2. The extinction of the Deccan kingdoms was a great political victory for the Mughals. Which of the statements given above is/are correct? a) 1 only b) 2 only c) Both 1 and 2 d) Neither 1 nor 2

Q204. Which of the following was/were parts of Mughal styles of architecture? 1. Charbagh style 2. Pietra dura ornamentation 3. Use of Red sandstone Select the correct answer using the code given below: a) 2 only b) 1 and 3 only c) 1 and 2 only d) 1, 2 and 3

Q205. Consider the following statements: 1. was the first Mughal ruler who undertook construction on a large scale. 2. The tradition of Mosque-building reached its climax under Shah Jahan. Which of the statements given above is/are correct? a) 1 only b) 2 only c) Both 1 and 2 d) Neither 1 nor 2

Q206. Daswant and Basawan were two famous painters in the royal court of a) Jahangir b) c) Akbar d) Humayun

Q207. With reference to the progress of painting during the reign of Jahangir, consider the following statements: 1. Significant progress was made in portrait painting and paintings of animals during his tenure. 2. Mansur was famous for portrait painting in this period. 3. European painting was introduced by the Portuguese priests during his reign. Which of the statements given above is/are correct?

©Jatin Verma All Rights Reserved. https://www.jatinverma.org P a g e | 46

a) 2 only b) 1 and 2 only c) 3 only d) 1 and 3 only

Q208. Consider the following statements: 1. During the time of Aurangzeb, there was a dispersal of artists to different places in the country. 2. The successors of Aurangzeb were not able to revive the . Which of the statements given above is/are correct? a) 1 only b) 2 only c) Both 1 and 2 d) Neither 1 nor 2

Q209. With reference to the development of under the Mughals, consider the following statements: 1. The literary activities were confined to the court and the nobles during this period. 2. There was no significant development of regional languages due to a lack of royal patronage. Which of the statements given above is/are correct? a) 1 only b) 2 only c) Both 1 and 2 d) Neither 1 nor 2

Q210. With reference to the administrative system of the Mughals, the term 'Khalisa' was used to denote a) Properties of newly occupied territories. b) Cities having ports or connectivity through inland waterways. c) Lands kept and managed by the imperial revenue department or directly by the emperor. d) Large urban centres administrated independent governors.

©Jatin Verma All Rights Reserved. https://www.jatinverma.org P a g e | 47

Answer Key

Qs. Ans. Qs. Ans. Qs. Ans. Qs. Ans. Qs. Ans. 1 C 11 D 21 B 31 C 41 B 2 D 12 C 22 D 32 D 42 D 3 A 13 C 23 C 33 A 43 C 4 D 14 D 24 B 34 B 44 C 5 A 15 D 25 B 35 B 45 D 6 C 16 C 26 C 36 C 46 B 7 D 17 D 27 B 37 C 47 D 8 D 18 C 28 D 38 C 48 C 9 D 19 C 29 B 39 C 49 A 10 C 20 C 30 D 40 C 50 A

Qs. Ans. Qs. Ans. Qs. Ans. Qs. Ans. Qs. Ans. 51 B 61 C 71 A 81 C 91 B 52 C 62 A 72 A 82 D 92 C 53 D 63 B 73 D 83 C 93 A 54 C 64 B 74 A 84 B 94 D 55 C 65 A 75 C 85 B 95 C 56 C 66 D 76 C 86 B 96 C 57 D 67 D 77 A 87 B 97 B 58 C 68 B 78 D 88 D 98 D

©Jatin Verma All Rights Reserved. https://www.jatinverma.org P a g e | 48

59 D 69 D 79 D 89 C 99 C 60 B 70 C 80 C 90 A 100 C

Qs. Ans. Qs. Ans. Qs. Ans. Qs. Ans. Qs. Ans. 101 D 111 C 121 D 131 C 141 B 102 B 112 A 122 D 132 A 142 C 103 B 113 B 123 B 133 B 143 D 104 A 114 A 124 C 134 D 144 B 105 C 115 A 125 B 135 B 145 B 106 C 116 B 126 B 136 C 146 D 107 A 117 D 127 D 137 A 147 D 108 D 118 A 128 D 138 C 148 D 109 A 119 D 129 D 139 D 149 C 110 A 120 C 130 A 140 D 150 B

Qs. Ans. Qs. Ans. Qs. Ans. Qs. Ans. Qs. Ans. 151 B 161 D 171 C 181 B 191 C 152 B 162 D 172 A 182 D 192 D 153 C 163 B 173 B 183 D 193 C 154 D 164 D 174 D 184 B 194 C

©Jatin Verma All Rights Reserved. https://www.jatinverma.org P a g e | 49

155 A 165 A 175 D 185 A 195 D 156 D 166 B 176 A 186 C 196 D 157 B 167 A 177 C 187 B 197 A 158 D 168 D 178 B 188 A 198 D 159 C 169 C 179 B 189 C 199 D 160 B 170 C 180 B 190 A 200 D

Qs. Ans. 201 D 202 C 203 D 204 D 205 B 206 C 207 B 208 A 209 D 210 C

©Jatin Verma All Rights Reserved. https://www.jatinverma.org P a g e | 50

Explanation

1. The "Black Hole Tragedy" was associated with the British conquest of Bengal. Siraj-Ud-Daulah, the Nawab of Bengal, captured Fort William from English in 1756. The British prisoners of war were lodged in a prison room in the fort. Out of the 146 white prisoners, only 23 survived the next morning when the prison room was opened, the rest having trampled one other down for places near the window. Excessive heat and suffocation took a heavy toll. Source) Chapter 5; Page number 96

2. In 18 century, Bengal made unprecedented progress during the regional rulers i.e. Murshid Quli, Shujauddin, Sarfaraz Khan, Alivardi Khan. Under the rule of these rulers, Bengal made unprecedented progress. There were other factors too, which made Bengal prosperous, for instance, the rest of India was disturbed by inter-border disputes, the Maratha invasions, Jat revolts, and external invasions by Nadir Shah and Ahmed Shah Abdali. The region of Bengal was fortunate enough to escape these challenges. Source) Chapter 5; Page number 94

3. Statement 1 is correct. concluded two important treaties at Allahabad in August 1765—one with the and the other with the Mughal Emperor, Shah Alam II. Nawab Shuja-ud-Daula agreed to: (i) surrender Allahabad and Kara to Emperor Shah Alam II; Hence, statement 2 is incorrect. (ii) pay Rs 50 lakh to the Company as war indemnity; and (iii) give Balwant Singh, Zamindar of Banaras, full possession of his estate. Shah Alam II agreed to: (i) reside at Allahabad, to be ceded to him by the Nawab of Awadh, under the Company’s protection; (ii) issue a farman granting the diwani of Bengal, Bihar and Orissa to the East India Company in lieu of an annual payment of Rs 26 lakh; hence, statement 1 is correct. (iii) a provision of Rs 53 lakh to the Company in return for nizamat functions (military defence, police, and administration of justice) of the said provinces. Source) Chapter 5; page number 100

4. All of the mentioned factors were responsible for the success of the British forces in India. Superior Arms, Military and Strategy: The firearms used by the English, which included muskets and cannons, were better than the Indian arms both in the speed of firing and in range. Better Military Discipline and Regular Salary: A regular system of payment of salaries and a strict regime of discipline were the means by which the English Company ensured that the officers and the troops were loyal. Civil Discipline and Fair Selection System: The Company officers and troops were given a charge on the basis of their reliability and skill and not on hereditary or caste and clan ties. They themselves were subject to strict discipline and were aware of the objectives of their campaigns. Brilliant Leadership and Support of Second Line Leaders: Clive, Warren Hastings, Elphinstone, Munro, Marquess of Dalhousie, etc., displayed rare qualities of leadership. The English also had the

©Jatin Verma All Rights Reserved. https://www.jatinverma.org P a g e | 51

advantage of a long list of secondary leaders like Sir Eyre Coote, Lord Lake and Arthur Wellesley who fought not for the leader but for the cause and the glory of their country. Strong Financial Backup: The income of the Company was adequate enough to pay its shareholders handsome dividends as also to finance the English wars in India. Furthermore, England was earning fabulous profits from its trade with the rest of the world. Nationalist Pride: An economically thriving British people believing in material advancement and proud of their national glory faced the ‘weak, divided-amongst-themselves Indians’ bereft of a sense of unified political nationalism. The lack of materialistic vision among Indians was also a reason for the success of the English Company. Source) Chapter 5

5. Statement 1 is incorrect. Guru Gobind Singh transformed the Sikhs into a militant sect in defence of their religion and liberties. Statement 2 is incorrect. In the wake of the invasions of Nadir Shah and Ahmad Shah Abdali, the Sikhs once again asserted their authority. At this stage, they organized themselves into 12 misls or confederacies which exercised control over different parts of the kingdom. Ranjit Singh was the son of Mahan Singh, the leader of the Sukarchakiya misl. Source) Chapter 5; Page number 78

6. Statement 1 is correct. The treaty of Salbai was signed between Warren Hastings and Mahadaji . As per the agreement, Salsette and Bassein were given to the British. Raghunath Rao was pensioned off and Madhav Rao II was accepted as the Peshwa. It provided the British twenty years of peace with the Marathas. Statement 2 is correct. The treaty of Salbai enabled the British to exert pressure on Mysore with the help of the Marathas in recovering their territories from Haider Ali. Source) Chapter 5; page number 112

7. The princely states of Satara (1848) and Jhansi & Nagpur (1854) were annexed under the Doctrine of Lapse. Lord Dalhousie annexed Awadh in 1856 after deposing Nawab on grounds of misgovernment. Source) Chapter 5; page number 135

8. The Third Anglo-Mysore war ended with the Treaty of Seringapatam in 1792. o As per the treaty, Tipu had to cede half of his kingdom to the English including the areas of Malabar, Dindigul, Coorg and Baramahal. o He also had to pay Rs.3 Crore as war indemnity to the British. o Tipu also had to surrender two of his sons as surety to the British till he paid his due.  Tipu Sultan of Mysore refused to enter into Subsidiary alliance, but after the British victory in the Fourth Anglo-Mysore War, Mysore was forced to become a subsidiary state in 1799.

©Jatin Verma All Rights Reserved. https://www.jatinverma.org P a g e | 52

9. First Anglo-Mysore War (1767 - 69): In the first Anglo-Mysore War, the Marathas, Nizam of and British together fought against Haidar Ali. In 1769, the Treaty of Madras was signed which brought an end to the war. Second Anglo-Mysore War (1780-84): The treaty of 1769, served more like a truce and Haider Ali accused the British of not observing the terms of this treaty. In the second Anglo-Mysore war the Marathas, Nizam together with Haidar Ali fought against the British. Second Anglo- Mysore War ended by signing a Treaty of Mangalore in 1784 by Tipu Sultan. Third Anglo-Mysore War (1790-92): The Marathas, Nizam and British (Triple Alliance) together fought against Tipu Sultan in the Third Anglo-Mysore War. Treaty of Srirangapatnam was signed by Tipu in 1792. The terms of the Treaty were dictated by the British.

10. The Second Anglo–Mysore War was a conflict between the Kingdom of Mysore and the British East India Company from 1780 to 1784. The Second Anglo-Maratha War was the second conflict between the British East India Company and the Maratha Empire in India from 1803–1805. The First Anglo-Burmese War, also known as the First Burma War, was the first of three wars fought between the British and Burmese empires from 5 March 1824 – 24 February 1826. The First Anglo-Sikh War was fought between the and the East India Company in 1845 and 1846.

11. Statement 1 is correct: The doctrine of lapse was the pro-imperialist approach to expand the realm of the British Kingdom in India. According to this doctrine, any under the direct or indirect (as a vassal) control of the East India Company where the ruler did not have a legal male heir would be annexed by the company. Statement 2 is Incorrect: The Doctrine of Lapse was not introduced by Lord Dalhousie even though it was he who documented it and used it widely to acquire territories for the British. Statement 3 is correct: After Dalhousie became the Governor-General, in 1848, Satara was acquired as per this doctrine. Other major states annexed by the British as per the Doctrine of Lapse: o Jaipur – 1849 o Sambalpur – 1849 o – 1852 o Jhansi – 1853 o Nagpur – 1854

12. Both statements 1 and 2 is Incorrect: Bajirao I, considered the greatest of all the , had started a confederacy of prominent Maratha chiefs to manage the rapidly expanding Maratha power. Under the arrangement of the Maratha confederacy, each prominent family under a chief was assigned a sphere of influence which he was supposed to conquer and rule, but in the name of the then Maratha King, Shahu. The Maratha families which emerged prominently were- 1. The Gaekwad of Baroda 2. The Bhonsle of Nagpur 3. The Holkars of Indore 4. The Sindhias of Gwalior 5. The Peshwas of Poona

©Jatin Verma All Rights Reserved. https://www.jatinverma.org P a g e | 53

13. Both statements 1 and 2 is Incorrect: Bajirao I, considered the greatest of all the Peshwas, had started a confederacy of prominent Maratha chiefs to manage the rapidly expanding Maratha power. Under the arrangement of the Maratha confederacy, each prominent family under a chief was assigned a sphere of influence which he was supposed to conquer and rule, but in the name of the then Maratha King, Shahu. The Maratha families which emerged prominently were- 1. The Gaekwad of Baroda 2. The Bhonsle of Nagpur 3. The Holkars of Indore 4. The Sindhias of Gwalior 5. The Peshwas of Poona

14. All of the statements given above are correct:  The Third Battle of Panipat took place on 14 January 1761 at Panipat, between Maratha Empire and invading Afghan army of under the King of Afghans, Ahmad Shah Abdali, supported by three Indian allies—the Najib-ud-daulah, Afghans of the Doab region, and Shuja-ud- Daula-the Nawab of Awadh. In this Battle army of Ahmed Shah Abdali defeated Marathas army.  The Fourth Anglo–Mysore War was a conflict in between the Kingdom of Mysore against the British East India Company and the Hyderabad Deccan in 1798–99. This was the final conflict of the four Anglo–Mysore Wars. The British captured the capital of Mysore. The ruler Tipu Sultan was killed in the battle.  British hatched a conspiracy by which they planned to place Mir Jafar on the throne of Bengal. They first presented a near impossible set of demands which Siraj-ud-Daulah refused to comply with thereby worsening the already tense situation. A battle ensued at Plassey, (20 miles from present Murshidabad) in June 1757 AD. Mir Jafar and Rai Durlabh betrayed the Nawab and sided with the British. Nawab Siraj-ud-Daulah was put to death by Mir Jafar’s son Miran. The British emerged winner in the battle of Plassey.  15. Tipu Sultan established embassies in France, Turkey and Egypt on modern lines.Tipu Sultan, the Tiger of Mysore/Tipu Sahib, is known for his valiant acts in several wars and the sacrifices he made to save his land from foreign invaders. He is remembered for saving Deccan India from the British for a long period of time. He was the ruler of the Kingdom of Mysore from 1782 to 1799.

16. The Battle of Porto Novo was fought on 1 July 1781 between forces of the Kingdom of Mysore and British East India Company in the place called Porto Novo (now known as Parangipettai), during the Second Anglo-Mysore War. Battle of Wandiwash was the Third Carnatic War fought between the French and the British. The Battle of Calicut was a series of engagements that took place between 7 and 12 December 1790 near the port of Calicut on the of India, during the Third Anglo-Mysore War. The Battle of Calicut was fought between the Mysore Kingdom and East India Company. The , also known as or The Battle of Ganeshkhind, took place at modern-day Khadki, India on 5 November 1817 between the forces of the British East India Company and the Maratha Empire under the leadership of Baji Rao II.

©Jatin Verma All Rights Reserved. https://www.jatinverma.org P a g e | 54

17. Statement 1 is Incorrect: War indemnity of more than 1 crores of Rupees was to be given to the English. Statement 2 and 3 is Correct: The Jalandhar Doab between the Beas and the Sutlej was annexed to the Company’s dominions. The strength of the Sikh army was reduced. Statement 4 is Incorrect: Under the Treaty of Bhairowal The Regent, Maharani Jindan Kaur, mother of the , was awarded an annual pension of 150,000 rupees and replaced by a Council of Regency composed of leading Chiefs and Sirdars acting under the control and guidance of the British Resident.

18. Anglo-Nepalese Relations-Treaty of Sagauli Anglo-Burmese Relations- Treaty of Yandabo Anglo-Tibetan Relations-Treaty of Lhasa

19. Statement 1 and 2 is Incorrect: The Sannyasi rebellion or Sannyasi Revolt (1770-1820) were the activities of sannyasis and fakirs (Hindu ascetics, respectively) in Bengal against the East India Company rule in the late 18th century. It is also known as the Sannyasi rebellion which took place around Murshidabad and Baikunthupur forests of Jalpaiguri. This rebellion was centred in the forests of Murshidabad and Baikunthupur, Bengal. Equal participation of Hindus and Muslims Characterised the uprisings sometimes referred to as the Fakir Rebellion. Statement 3 is Correct: Bankim Chandra Chatterjee’s novel Anandamath, written in 1882, was set in the backdrop of the Sanyasi Rebellion. This book was banned by the British. India’s National Song ‘Vande Mataram’ was taken from this novel.

20. Polygars (Palaiyakkarars) were feudal lords who were appointed as military chiefs and administrative governors from the time of the Vijayanagara Empire in parts of Southern India. (They were given the charge of a Palayam or a group of villages). It was the Polygars who collected taxes from the cultivators. But the East India Company came into conflict with the Polygars over the question of who should collect taxes, and sought to control the Polygars. The first rebellion, also called the First Polygar War broke out in September 1799 in Tirunelveli district in modern Tamil Nadu.The Polygars were led by Kattabomma Nayak (also called Veerapandi Kattabomman) who was in charge of Panchalankurichi Palayam. The main centres of Poligars uprising were Tirunelveli, Ramanathapuram, Sivaganga, Sivagiri, Madurai and North Arcot.

21. Statement 1 is Correct: The Paika Rebellion also called the Paika Bidroha. It was an armed rebellion against the British East India Company's rule in Odisha in 1817. The Paikas rose in rebellion under their leader Bakshi Jagabandhu and, projecting Jagannath as the symbol of Odia unity, Statement 2 is Incorrect: The Paikas were the traditional land-owning militia of Odisha and served as warriors. When armies of the East India Company overran most of Odisha in 1803, the Raja of Khurda lost his primacy and the power and prestige of the Paikas went on a decline.

22. Statement 1 and 2 is Correct: The Kukas, also called Namdharis, were a sect within Sikhism.They started out as a group for religious purification in Sikhism but under Ram Singh, the movement acquired a political overtone with the established aim of restoring the Sikh rule in

©Jatin Verma All Rights Reserved. https://www.jatinverma.org P a g e | 55

Punjab and ousting the foreign powers. The principles of Swadeshi and non-co-operation, which Mahatma Gandhi introduced in our freedom movement, were expounded by Guru Ram Singh for the Namdharis. The Guru’s Non-cooperation Movement was based on a few things such as the boycott of education institutions of British and laws established by them. They were rigid in their clothing and wore only hand-spun white attire.

23. Option C is the Correct answer: The Sannyasi rebellion or Sannyasi Revolt (1770-1820) were the activities of sannyasis and fakirs (Hindu ascetics, respectively) in Bengal against the East India Company rule in the late 18th century. The Moamoria rebellion (1769–1805) was the 18th-century conflict in Assam between the Moamorias, who were mainly Motok (Chutiyas and Moran) and Kachari adherents of the Moamara Sattra, and the Ahom kings. The Paika Rebellion also called the Paika Bidroha. It was an armed rebellion against the British East India Company's rule in Odisha in 1817. Ahom’s Revolt took place in Assam region after the First Anglo Burmese War. The rebellion was against the annexation of the territories under British Empire in 1828. 24. The correct chronological sequence of the events is: 1. Revolts in the British Indian Army—in Bengal (1764) 2. Vellore (1806) 3. Barrackpore (1825) mutiny 4. Afghan Wars (1838-42) Source) Chapter 7; page number 188

25. Statement 1 is incorrect. During the revolt of 1857, the sepoys were accompanied by a rebellion of the civil population, particularly in the north-western provinces and Awadh. Their accumulated grievances found immediate expression and they rose en masse to give vent to their opposition to British rule. Statement 2 is correct. There was widespread participation in the revolt by the peasantry, artisans, shopkeepers, day labourers, zamindars, religious mendicants, priests and civil servants which gave it real strength as well as the character of a popular revolt. The peasants and petty zamindars gave free expression to their grievances by attacking the money-lenders and zamindars who had displaced them from the land. They took advantage of the revolt to destroy the money-lenders' account books and debt records. They also attacked the British-established law courts, revenue offices (tehsils), revenue records and police stations. Source) Chapter 7; Page number

26. Statements 2, 3 and 4 are correct. The revolt of 1857 marks a turning point in the history of India. It led to changes in the system of administration and the policy of the Government. As a result of the revolt, the era of annexation and expansion ended and the British promised to respect the dignity and rights of the native princes. Hence, statement 1 is incorrect.

27. The reform movements could broadly be classified into two categories—the reformist movements and the revivalist movements. The Brahmo Samaj, the Prarthana Samaj, the Aligarh Movement were

©Jatin Verma All Rights Reserved. https://www.jatinverma.org P a g e | 56

reformist movements while the Arya Samaj and the Deoband movement were the revivalist movements like. Source) Chapter 8; page number 215

28. All the statements are correct. The colonial policies of the East India Company destroyed the traditional economic fabric of Indian society. Due to the highly unpopular revenue settlement of East India Company, the peasants resorted to loans from money-lenders/traders at usurious rates, the latter often evicting the former from their land on non-payment of debt dues. These money-lenders and traders emerged as the new landlords, while the scourge of landless peasantry and rural indebtedness has continued to plague Indian society to this day. The older system of zamindari was forced to disintegrate. For the artisans and handicrafts people, the annexation of Indian states by the Company cut off their major source of patronage the native rulers and the nobles. The native rulers and the nobles could not now afford to be patrons of the crafts workers. British policy discouraged Indian handicrafts and promoted British goods. The highly skilled Indian craftsmen were forced to look for alternate sources of employment that hardly existed, as the destruction. British imposed high tariff duties on Indian-made goods. At the same time, the import of British goods into India attracted low tariffs, thus encouraging their entry into India. The traditional landed aristocracy or the Zamindars often saw their land rights forfeited with frequent use of a quo warranto by the administration. This resulted in a loss of status for them in the villages. In Awadh, the storm centre of the revolt, 21,000 taluqdars had their estates confiscated. Source) Chapter 7; Page number 186

29. The Religious Disabilities Act, 1856, which modified Hindu customs, for instance declaring that a change of religion did not debar a son from inheriting the property of his heathen father. Source) Chapter 7; page number

30. The most ferocious battles were fought in Delhi, Awadh, Rohilkhand, Bundelkhand, Allahabad, , Meerut and western Bihar. The rebellious forces under the commands of Kanwar Singh in Bihar and Bakht Khan in Delhi gave a stunning blow to the British. In Kanpur, Nana Sahib was proclaimed as the Peshwa and the brave leader Tantya Tope led his troops. Rani Lakshmibai was proclaimed the ruler of Jhansi who led her troops in the heroic battles with the British. The limited territorial spread was one factor; there was no all-India veneer about the revolt. The eastern, southern and western parts of India remained more or less unaffected. This was probably because the earlier uprisings in those regions had been brutally suppressed by the Company. The revolt began at Meerut on May 10, 1857 and then, gathering force rapidly, soon embraced a vast area from the Punjab in the north and the Narmada in the south to Bihar in the east and Rajputana in the west. Source) Chapter 7; page number, Page number 194/95 https://www.india.gov.in/people-groups/life- cycle/kids/indian-freedom-struggle-1857-1947

31. Statement 1 is correct. In 1856 Lord Canning's Government passed the General Service Enlistment Act.

©Jatin Verma All Rights Reserved. https://www.jatinverma.org P a g e | 57

Statement 2 is correct. The General Service Enlistment Act which decreed that all future recruits to the Bengal Army would have to give an undertaking to serve anywhere their services might be required by the Government. This caused resentment among the sepoys. Source) Chapter 7; Page number 178

32. All of the given statements are Correct: The Native Marriage Act passed in 1872 legalised unorthodox inter-caste marriage. The Native Marriage Act signified legislative action in prohibiting child marriage. The Age of Consent Act, 1891, was legislation enacted in British India on 19 March 1891 which raised the age of consent for sexual intercourse for all girls, married or unmarried, from ten to twelve years in all jurisdictions, its violation subject to criminal prosecution as rape. The Child Marriage Restraint Act, 1929, passed on 28 September 1929, in the Imperial Legislative Council of India, fixed the age of marriage for girls at 14 years and boys at 18 years which was later amended to 18 for girls and 21 for boys.

33. Pair 1 is Correct: Bharat Stree Mahamandal was the first women's organisation in India founded by Chaudhurani in Allahabad in 1910. Pair 2 is Incorrect: Arya Mahila Samaj was founded by Pandita Ramabai Saraswati to provide a support network for newly educated women. Pair 3 is Incorrect: Ramabai Ranade founded the Ladies Social Conference (Bharat Mahila Parishad), under the parent organisation National Social Conference, in 1904 in Bombay

34. Statement 1 is Correct: Jagannath Shankar Seth was active in the educational field. In 1856, the British Government had announced some grant in aid to private institution but Nana taking advantage of this started an English Marathi school in Girgaum. He contributed funds to the girl’s school started by Student’s Literary and Scientific Society, in spite of strong opposition from the Hindu community. Statement 2 is Incorrect: Karsondas Mulji started Satyaprakash, a weekly in Gujarati, in which he attacked what he perceived to be the immoralities of the or hereditary high priests of the Pushtimarg . Statement 3 is Correct: Vidyasagar pressed on and urged the British to pass legislation that will allow Hindu widows to remarry. To support his request, Ishwar Chandra Vidyasagar collected almost 1000 signatures and sent his petition to the Indian legislative council. The council received thousands of signatures for and against this measure but the members finally decided to support the enlightened minority, The Hindu widow remarriage act was passed in 1856.

35. Statement 1 is Correct: At Delhi, the symbolic leadership was to the Mughal emperor, Bahadur Shah, but the real command was led by General Bakht Khan. Statement 2 is Incorrect: Begum Hazrat Mahal took over the reign of Lucknow and Birjis Qadir, her son, was declared Nawab. Henry Lawrence, the British resident, was killed. The remaining Europeans were evacuated by Sir Colin Campbell, the new commander - in - chief. Statement 3 is Correct: Kanpur rose under Nana Saheb, Tantia Tope, Azimullah Khan. Sir Hugh Wheeler, commanding the station, surrendered. Nana Saheb declared himself the Peshwa and Bahadhur Shah as

©Jatin Verma All Rights Reserved. https://www.jatinverma.org P a g e | 58

Statement 4 is Correct: Rani Laxmibai, the most outstanding leader of the revolt, was driven out of Jhansi with the application of Lapse's Doctrine as Lord Dalhousie, the Governor-General refused to allow her adopted son to succeed to the throne.

36. 1857 revolt came to be interpreted as a “planned war of national independence”, by V.D. Savarkar in his book, The Indian War of Independence,1857. Savarkar called the revolt the “Firsts War of Independence”.

37. One of the famous revolt in the history of India which occurred in the governor-generalship of Lord Canning in the year 1857.

38. Statement 1 and 3 is Correct: The objects of Queen Victoria’s Proclamation in 1858 were to disclaim any intention to annex the Indian States as the announcement reversed Lord Dalhousie’s pre-war policy of political unification through princely state annexation. It was also to place the Indian administration under the British Crown. Statement 2 is Incorrect: However, it was not to regulate East India Company’s trade with India.

39. Statement 1 is Incorrect: Narayana Guru was a spiritual leader and social reformer in India. Born into a family of the Ezhava caste. He led a reform movement in Kerala, against the injustice in the caste- ridden society of Kerala in order to promote spiritual enlightenment and social equality. Statement 2 is Correct: Bahishkrit Hitakarini Sabha is a central institution formed by Dr Babasaheb Ambedkar for removing difficulties of the untouchables and placing their grievances before the government. Statement 3 is Incorrect: Erode Venkatappa Ramasamy commonly known as Periyar, also referred to as Thanthai Periyar, was an Indian social activist, and politician who started the Self-Respect Movement and Dravidar Kazhagam.

40. The Religious Disabilities Act of 1850 was a law passed in British India under East India Company rule that abolished all laws affecting the rights of persons converting to another religion or caste. The new Act allowed Indians who converted from one religion to other religion equal rights under no law, especially in the case of inheritance.

41. was founded as a secular Native Female School (for the secular education of girls) in 1849 by John Elliot Drinkwater Betune. In 1854, Charles Wood prepared a despatch on an educational system for India. This document is considered as the "Magna Carta of English Education in India". The National Council of (NCWI) was formed in 1925. The Indian Women’s University set up by Professor D.K.Karve in 1916 was one of the outstanding institutions imparting education to women.

42. The Revolt of 1857 was an extremely important event in Indian history. It was more a product of but was accumulated grievances of the people against the Company’s administration and of their dislike for the foreign regime. The control of Indian administration was passed on to the British Crown by

©Jatin Verma All Rights Reserved. https://www.jatinverma.org P a g e | 59

the Government of India Act, 1858 which resulted in the withdrawal of the Doctrine of lapse and the beginning of a new rule which means the end of the Mughal rule as Peshwa rule.

43. The Servants of India Society was formed in Pune, Maharashtra, on June 12, 1905, by Gopal Krishna Gokhale, who left the Deccan Education Society to form this association. Along with him were a small group of educated Indians, as Natesh Appaji Dravid, Gopal Krishna Deodhar and Anant Patwardhan who wanted to promote social and human development and overthrow the British rule in India. The Society organized many campaigns to promote education, sanitation, health care and fight the social evils of untouchability and discrimination, alcoholism, poverty, oppression of women and domestic ab

44. Raja Rammohan Roy founded the Brahmo Sabha in 1828 which later came to be known as Brahmo Samaj. In 1814, Ram Mohan Roy set up the Atmiya Sabha (or Society of Friends) in Calcutta to propagate the monotheistic ideals of the and to campaign against , caste rigidities, meaningless rituals, and other social ills. The Tattvabodhini Sabha ( Propagating/Searching Society) was a group started in Calcutta on 6 October 1839 as a splinter group of the Brahmo Samaj, reformers of Hinduism and Indian Society. Hence, the correct answer is C. Source) Chapter 9; page number 228

45. The Prarthana Samaj was established in Bombay by Dr. Atma Ram Pandurang (1825- 1898) in 1876 with the objective of rational worship and social reform. Mahadeo Govind Ranade (1842-1901) joined the samaj in 1870 devoted his entire life to Prarthana Samaj. Keshab Chandra Sen helped Atmaram Pandurang in founding the Prarthana Samaj. Other leaders of the samaj were R.G. Bhandarkar (1837- 1925) and N.G. Chandavarkar (1855-1923). Source) Chapter 9; 234

46. Paramahansa Sabha or Paramhans Mandali was a precursor of the Prarthana Samaj. It was something like a secret society to spread liberal ideas and encourage the breakdown of caste and communal barriers. In Bombay, the Paramhans Mandali was founded in 1840 to work for the abolition of caste. Many of these reformers and members of reform associations were people of upper castes. Often, in secret meetings, these reformers would violate caste taboos on food and touch, in an effort to get rid of the hold of caste prejudice in their lives. Source) Chapter 9; Page number 234, http://www.ncert.nic.in/ncerts/l/hess203.pd

47. Both the statements are incorrect. The SNDP movement was an example of a regional movement born out of the conflict between the depressed classes and upper castes. It was started by Sree Narayana Guru Swamy (1856-1928) among the Ezhavas of Kerala, who were a backward caste of toddy-tappers and were considered to be untouchables, denied education and entry into . The Ezhavas were the single largest caste group in Kerala constituting 26 per cent of the total population. Narayana Guru, himself from the Ezhava caste, took a stone from the Neyyar river and installed it as a Sivalinga at Aruvippuram on Sivaratri in 1888. Source) Chapter 9; Page number 248

©Jatin Verma All Rights Reserved. https://www.jatinverma.org P a g e | 60

48. Both the statements are correct. The Indian Association of Calcutta (also known as the Indian National Association) was the most important of pre- Congress associations and aimed to “promote by every legitimate means the political, intellectual and material advancement of the people.” It set out to: (i) create a strong public opinion on political questions, and (ii) unify Indian people in a common political programme. The Zamindari Association, more popularly known as the ‘Landholders’ Society’, was founded to safeguard the interests of the landlords. Although limited in its objectives, the Landholders’ Society marked the beginning of an organised political activity and use of methods of constitutional agitation for the redressal of grievances. Source) Chapter 10; page number 271

49. Statement 1 is correct. Most of the political associations in the early half of the nineteenth century were dominated by wealthy and aristocratic elements. The political associations formed after 1850, especially those that came into being in the 1870s and 1880s, were led by English-educated professionals such as lawyers. The more important ones were the Poona Sarvajanik Sabha, the Indian Association, the Madras Mahajan Sabha, the Bombay Presidency Association. Statement 2 is incorrect. They worked with the idea that the people should be sovereign – a modern consciousness and a key feature of nationalism. In other words, they believed that the Indian people should be empowered to take decisions regarding their affairs. Some of the important campaigns were: (i) for the imposition of import duty on cotton (1875) (ii) against Lytton’s Afghan adventure (iii) against Arms Act (1878) (iv) against Vernacular Press Act (1878) Source) Chapter 10; page number 273, http://www.ncert.nic.in/ncerts/l/hess205.pdf

50. The Madras Mahajan Sabha was founded in 1884 by M. Viraraghavachari, B. Subramaniya Aiyer and P. Anandacharlu. Source) Chapter 10; page number 272

51. The East India Association was organized by Dadabhai Naoroji in 1866 in London to discuss the Indian question and influence public men in England to promote Indian welfare. Later, branches of the association were started in prominent Indian cities. Source) Chapter 10; page number 272

52. Option C is the Correct Answer: In 1917, the Madras Presidency Association was formed which demanded separate representation for the lower castes in the legislature.

53. Pair 1 is Incorrect: The Poona Sarvajanik Sabha was founded in 1867 by Mahadeo Govind Ranade, with the object of serving as a bridge between the government and the people Pair 2 is Correct: The Madras Mahajan Sabha was founded in 1884 by M.Viraraghvachari, B.Subramaniya Aiyer and P. Anandacharlu.

©Jatin Verma All Rights Reserved. https://www.jatinverma.org P a g e | 61

Pair 3 is Correct: The Indian National Association also known as The Indian Association of Calcutta superseded the Indian League and was founded in 1876 by Younger nationalists of Bengal led by Surendranath Banerjee and Ananda Mohan Bose

54. Ishwar Chandra Vidyasagar: Vidyasagar was a Bengali Sanskrit pundit, educator, social reformer, writer and philanthropist. He was one of the greatest intellectuals and activists of the 19th century. At the age of twenty-one, Ishwar Chandra joined the Fort William College as the head of the Sanskrit department. His study of ancient texts convinced him that the status of Hindu women of his time was not sanctioned by the scriptures and was because of the existing power relations in society. Betaal Panchavinsati (1847), Banglar Itihaas (1848), Jivancharita (1849), Shakuntala (1854), (1860), Seetar Vanavas (1860) are some of his works.

55. Both statements 1 and 2 are Correct: Narayan Malhar Joshi was an Indian trade union leader and follower of Gopal Krishna Gokhale founded the Social Service League in 1911. Shiv Narayan Agnihotri became an active member of the Brahmo Samaj in 1875 and worked tirelessly for various reformist movements started by it. He left Brahmo Samaj in 1887. He formed Dev Samaj in February 1887.

56. Statement 1 is Incorrect: Movement originated with the life and teachings of Mirza Ghulam Ahmad based on the principles of the universal religion of all humanity, opposing Jihad(sacred war against non-Muslims). Statement 2 is Incorrect: The most important movement for the spread of modern education and social reform among Muslims was started by Sir (1817-1898). The Aligarh Movement was the push to establish a modern system of education for the Muslim population of British India, during the latter decades of the 19th century. Statement 3 is Correct: The Deoband Movement was organised by the orthodox section among the Muslim ulema as a revivalist movement with the twin objectives of propagating pure teachings of the Quran and Hadis among Muslims and keeping alive the spirit of Jihad against the foreign rulers.

57. Statement 1 is Correct and Statement 2 is Incorrect: The Akali Movement also known as Gurudwara Reform Movement was an offshoot of the Singh Sabha Movement. It aimed at liberating the Sikh gurudwaras from the control of corrupt Udasi mahants. These mahants were a loyalist and reactionary lot, enjoying government patronage. Statement 3 is Correct: The Akali Movement was a regional movement but not a communal one.

58. also known as Truth-seekers' Society was a social reform society founded by in Pune, Maharashtra, on 24 September 1873. It espoused a mission of education and increased social rights and political access for underprivileged groups, focused especially on women, Shudras, and , in Maharashtra.

59. Option D is the Correct answer Brahma Samaj: Reformist movements Prarthana samaj: Reformist movements

©Jatin Verma All Rights Reserved. https://www.jatinverma.org P a g e | 62

Aligarh movement: Reformist movements Arya samaj: Revivalist movements

60. established the Tattvabodhini Sabha (1839) at Calcutta to propagate Rammohan Roy’s ideas.

61. was popularly known as ‘Lokahitawadi’. He made powerful rationalist attacks on Hindu orthodoxy and preached religious and social equality.

62. The movement started by Derozio was called the Young Bengal Movement and his followers were known as the Derozians. They condemned religious rites and rituals and pleaded for the eradication of social evils, and improvement in the condition of women. 63. The Western-educated progressive Parsis like Dadabhai Naoroji, J.B. Wacha, S.S. Bangali and Naoroji Furdonji founded the Rahanumai Mazdayasanan Sabha (Religious Reform Association) in 1851. The prime objective of the association was- “the regeneration of the social condition of the Parsis and the restoration of the Zoroastrian religion to its pristine purity”. Rast Goftar (Voice of Truth) was its weekly organ.

64. R.P. Dutt opined that the Indian National Congress was born out of a conspiracy to abort a popular uprising in India and the bourgeois leaders were a party to it. Source) Chapter 11; page number 275

65. Statement 1 is correct. The moderate political activity involved constitutional agitation within the confines of law and showed a slow but orderly political progress. The Moderate leaders believed that political connections with Britain were in India’s interest at that stage of history and that the time was not ripe for a direct challenge to the British rule. Statement 2 is incorrect. The extremist leaders like Lala Lajpat Rai believed in the ‘safety valve’ theory. There is a theory that Hume formed the Congress with the idea that it would prove to be a ‘safety valve’ for releasing the growing discontent of the Indians. Source) Chapter 11; page number 276 and 277

66. All the statements are correct. The moderates called for the separation of judicial from executive functions. They criticized the oppressive and tyrannical bureaucracy and the time consuming judicial system of the British. They were also critics of an aggressive foreign policy which resulted in the annexation of Burma, attack on Afghanistan and suppression of tribals in the North-West—all costing heavily for the Indian treasury. They called for an increase in expenditure on welfare (i.e., health, sanitation), education—especially elementary and technical—irrigation works and improvement of agriculture, agricultural banks for cultivators, etc. Demand for better treatment for Indian labour abroad in other British colonies, where they faced oppression and racial discrimination. Source) Chapter 11; Page number 281

©Jatin Verma All Rights Reserved. https://www.jatinverma.org P a g e | 63

67. Statement 1 is correct. As the question of spreading the movement on pan India level and the question of passive resistance divided INC which led to split in 1907. Statement 2 incorrect. In spite of the movement having its genesis in anti-partition protest in Bengal, the boycott movement spread to many parts of India. Statement 3 is correct. The social base of the national movements now extended to include a certain zamindari section, the lower middle class in the cities and small towns and school and college students on a massive scale. Women came out of their homes for the first time and joined processions and picketing. Source) Chapter 12; 29

68. In 1904, V. D. Sarvarkar organized Abhinav Bharat as a secret society of revolutionaries. Source) Chapter 13; page number 321

69. Statement 1 is correct. The Indian National Congress Benares session was presided over by G.K. Gokhale. Statement 2 is incorrect. The goal of the Indian National Congress as 'self-government or Swaraj like that of the United Kingdom or the Colonies' was declared at the 1906 session at Calcutta presided over Dadabhai Naroji. Statement 3 is incorrect. The Indian National Congress took up the Swadeshi call and the Banaras Session, 1905, presided over by G.K. Gokhale, supported the Swadeshi and Boycott Movement for Bengal. The militant nationalists led by Tilak, Bipin Chandra Pal, Lajpat Rai and Aurobindo Ghosh were, however, in favour of extending the movement to the rest of India and carrying it beyond the programme of just Swadeshi and boycott to a full-fledged political mass struggle. The aim was now Swaraj and the abrogation of partition had become the 'pettiest and narrowest of all political objects'. The moderates, by and large, were not as yet willing to go that far. Source) Chapter 12; 30

70. Statements 1 and 3 are correct. By the dawn of the twentieth century, a band of nationalist thinkers had emerged who advocated a more militant approach to political work. These included Raj Narain Bose, Ashwini Kumar Datta, Aurobindo Ghosh in Bengal; Vishnu Shastri Chiplunkar and Tilak in Maharashtra; and Lala Lajpat Rai in Punjab as the most outstanding representative of this school of thought. This school of thought was based on 1. Hatred for foreign rule; since no hope could be derived from it, Indians should work out their own salvation; 2. Swaraj as the main goal of national movement; 3. Adoption of direct political action; 4. Belief in the capacity of the masses to challenge the authority. Hence, statement 2 is incorrect. 5. The requirement of personal sacrifices. They thought that a true nationalist to be always ready for personal sacrifices.

71. Statement 1 is correct. Statement 2 is incorrect. The Indian National Congress was established when 72 delegates from all over the country met at Bombay in December 1885. A.O. Hume was a retired English civil servant who mobilized leading intellectuals of the time and, with their cooperation, organized the first session of the Indian National Congress at Gokuldas Tejpal Sanskrit College. Womesh Chandra Bonnerjee the first session of the Indian National Congress.

©Jatin Verma All Rights Reserved. https://www.jatinverma.org P a g e | 64

Source) Chapter 11; page number 274, http://www.ncert.nic.in/ncerts/l/hess205.pdf

72. Statement 1 is Correct: The for Indian Independence was established in 1915 by Virendranath Chattopadhyay, Bhupendranath Dutta, Lala Hardayal and others with the help of German foreign office under ‘Zimmerman Plan’. These aimed to mobilise the Indian settlers abroad to send volunteers and arms to India. Statement 2 is Incorrect: The Ghadar Party was an Indian revolutionary organisation primarily founded by Indians. The party was headquartered in San Francisco, .

73. Pair 1 is Correct: (Young India Society) was a secret society founded by Vinayak Damodar Savarkar and his brother Ganesh Damodar Savarkar in 1904. Pair 2 is Incorrect: Ajit Singh(Bhagat Singh Uncle) organised the extremist Anjuman I Mohisban I Watan in Lahore with its journal Bharat Mata. Pair 3 is Incorrect: The Indian Home Rule Society (IHRS) was an Indian organisation founded in London in 1905 that sought to promote the cause of self-rule in British India. The organisation was founded by , with support from a number of prominent Indian nationalists in Britain at the time, including Bhikaji Cama, Dadabhai Naoroji and S.R. Rana.

74. Statement 1 is Incorrect and Statement 2 is Correct: In October 1906, a group of Muslim elites called the Simla Deputation, led by the Aga Khan, met Lord and demanded separate electorates for the Muslims and representation in excess of their numerical strength in view of ‘the value of the contribution’ Muslims were made to the defence of the empire. The same group took over the Muslim League, initially floated by Nawab Salimullah of DAca along with Nawabs Mohsin-ul-Mulk and Waqar-ul-Mulk in December 1906.

75. Statement 1 is Incorrect: The First session of the INC was held in Bombay instead of Pune due to outbreak of Plague and it was attended by 72 delegates Statement 2 is Correct: It was presided over by W.c.Banerjee and A.O.Hume assumed the charge of General Secretary. Statement 3 is Correct: When the first session held in Dec 18845, the secretary of State for India was Lord Randolph Churchill

76. Pair 1 is Incorrect: In Calcutta Session of 1906, under the leadership of Dadabhai Naoroji, Congress adopted Swaraj as the goal of Indian People. Pair 2 and 3 is Correct: The 1907 Surat session was held at the bank of the Tapti river in Surat. The Surat session was presided by Dr Rash Behari Ghosh. The 1909 Lahore Session was presided by Madan Mohan Malviya.

77.  Yugantar the revolutionary weekly started(1906).  Kesari Marathi daily was started by Lokmanya Tilak(1881).  Mitra Mela merged with Abhinav Bharat(1904).  Bomb thrown at Viceroy Hardinge by Rasbehari Bose and Sachin Sanyal(1912).

©Jatin Verma All Rights Reserved. https://www.jatinverma.org P a g e | 65

78. All of the statements given above are correct: 1. The social base of moderates was Zamindars and upper-middle classes in towns whereas the social base of extremists were middle and lower-middle classes in towns. 2. Moderates took ideological inspiration from European history whereas extremists took ideological inspiration from Indian History. 3. Moderates believed in England’s providential mission in India whereas Extremists rejected providential mission theory as an illusion. 4. Moderates believed political connections with Britain to be in India’s social, political and cultural interests whereas extremists believed that political connections with Britain would perpetuate British exploitation in India. 5. Moderates professed loyalty to the British crown whereas extremists believed that the British crown was unworthy of claiming Indian loyalty.

79. All of the statements given above are correct: 1. The social base of moderates was Zamindars and upper-middle classes in towns whereas the social base of extremists were middle and lower-middle classes in towns. 2. Moderates took ideological inspiration from European history whereas extremists took ideological inspiration from Indian History. 3. Moderates believed in England’s providential mission in India whereas Extremists rejected providential mission theory as an illusion. 4. Moderates believed political connections with Britain to be in India’s social, political and cultural interests whereas extremists believed that political connections with Britain would perpetuate British exploitation in India. 5. Moderates professed loyalty to the British crown whereas extremists believed that the British crown was unworthy of claiming Indian loyalty.

80. Statement 1 is Incorrect: Bal Gangadhar Tilak was not a founder member of the Indian National Congress. He joined INC in 1890. Statement 2 is Correct: B.G.Tilak was one of the first and strongest advocates of Swaraj and a strong radical in ‘Indian consciousness. He is known for his quote in Marathi: “Swarjya is my birthright and I shall have it” Statement 3 is Correct: He was among the first to advocate bringing the lower middle classes, the peasants, artisans and workers into the Congress fold.

81. Pair 1 is Incorrect: Khudiram, along with , attempted to assassinate a British judge, Magistrate Douglas Kingsford, by throwing bombs on the carriage they suspected the man was in. Magistrate Kingsford, however, was seated in a different carriage, and the throwing of bombs resulted in the deaths of two British women. Pair 2 is Incorrect: Anant Laxman Kanhere, a 17-year-old student of Aurangabad, shot Jackson on 21 December 1909 at a theatre where a drama was to stage in his honour on the eve of his transfer. Pair 3 is Correct: Hutatma Madan Lal Dhingra was an Indian revolutionary independence activist. While studying in England, he assassinated William Hutt Curzon Wyllie, a British official.

©Jatin Verma All Rights Reserved. https://www.jatinverma.org P a g e | 66

Pair 4 is Correct: The , also known as the Delhi-Lahore Conspiracy, refers to a conspiracy in 1912 to assassinate the then Viceroy of India, Lord Hardinge, on the occasion of transferring the capital of British India from Calcutta to . Hatched by the Indian revolutionary underground in Bengal and Punjab and headed by , the conspiracy culminated on the attempted assassination on 23 December 1912 when a homemade bomb was thrown into the Viceroys’ Howdah when the ceremonial procession moved through the Chandni Chowk suburb of Delhi.

82. Chittagong armoury raid, also known as the Chittagong uprising, was an attempt on 18 April 1930 to raid the armoury of police and auxiliary forces from the Chittagong armoury in the of by armed Indian independence fighters led by Surya Sen.

83. Important events during Lord Lytton's time (1876-1880) o Famine of 1876-78 affecting Madras, Bombay, Mysore, Hyderabad, parts of central India and Punjab. appointment of Famine Commission under the presidency of Richard Strachey(1878). o Royal Titles Act (1876), Queen Victoria assumed the title of Kaiser-i-Hind or Queen Empress of India. o The Vernacular Press Act (1878) o The Arms Act (1878) o The Second Afghan War (1878-80).

84. Statement 1 and 2 are Correct: The Bombay Presidency Association was started by Badruddin Tyabji, Pherozshah Mehta and K.T. Telang in 1885.He was the founding member of Indian national congress and the first Muslim who presided the annual session of congress.Third session of congress held in Madras(1887). He was not associated with Muslim league.

85. Statement 1 is incorrect. In the First World War (1914-1919), Britain allied with France, Russia, USA, Italy, and Japan against Germany, Austria-Hungary and Turkey. The Moderates supported the empire in the war as a matter of duty. The Extremists, including Tilak (who was released in June 1914), supported the war efforts in the mistaken belief that Britain would repay India’s loyalty with gratitude in the form of self-government. Statement 2 is correct. The revolutionaries decided to utilize the opportunity to wage a war on British rule and liberate the country. The revolutionary activity was carried out through the Ghadr Party in North America, Berlin Committee in Europe and some scattered mutinies by Indian soldiers, such as the one in Singapore. In India, for revolutionaries striving for immediate complete independence, the War seemed an opportunity, draining India of troops (the number of white soldiers went down at one point to only 15,000), and raising the possibility of financial and military help from Germany and Turkey-the enemies of Britain. Source) Chapter 14; page number 329

86. Statement 1 is correct. The Indian Home Rule movement was a movement in British India on the lines of Irish Home Rule movement and other home rule movements. Statement 2 is incorrect. Two Indian Home Rule Leagues represented the emergence of a new trend of aggressive politics. and BG Tilak were the pioneers of this new trend.

©Jatin Verma All Rights Reserved. https://www.jatinverma.org P a g e | 67

Source) Chapter 14; page number 329

87. Some of the factors leading to the formation of the Home Rule Movement were as follows: (i) A section of the nationalists felt that popular pressure was required to attain concessions from the government. Hence, statement 3 is incorrect. (ii) The Moderates were disillusioned with the Morley- Minto reforms. Hence, statement 2 is incorrect. (The Montagu–Chelmsford Reforms-1919) (iii) People were feeling the burden of wartime miseries caused by high taxation and a rise in prices, and were ready to participate in any aggressive movement of protest. Hence, statement 1 is correct. (iv) The war, being fought among the major imperialist powers of the day and backed by naked propaganda against each other, exposed the myth of white superiority. (v) Tilak was ready to assume leadership after his release in June 1914, and had made conciliatory gestures— to the government reassuring it of his loyalty and to the Moderates that he wanted, like the Irish Home Rulers, a reform of the administration and not an overthrow of the government. (vi) Annie Besant, the Irish theosophist based in India since 1896, had decided to enlarge the sphere of her activities to include the building of a movement for home rule on the lines of the Irish Home Rule Leagues. Source) Chapter 14; page number 330

88. All the statements are correct. The Rowlatt Act of 1919 allowed political activists to be tried without juries or even imprisoned without trial. It allowed the arrest of Indians without a warrant on the mere suspicion of 'treason'. Such suspects could be tried in secrecy without recourse to legal help. He called the Rowlatt Act the “Black Act” and argued that not everyone should get punishment in response to isolated political crimes. Gandhi called for a mass protest at all India level. But soon, having seen the constitutional protest meet with ruthless repression, Gandhi organized a Satyagraha Sabha and roped in younger members of Home Rule Leagues and the Pan Islamists. The forms of protest finally chosen included observance of a nationwide hartal (strike) accompanied by fasting and prayer, and civil disobedience against specific laws, and courting arrest and imprisonment. Source) Chapter 15; page number 358

89. Statement 1 is correct. Both the Moderates and the Extremists realized that the split had led to political inactivity. The extremists were welcomed back into Congress. The first Lucknow session was held in 1899. Statement 2 is incorrect. The Lucknow Congress was significant also for the famous Congress League Pact, popularly known as the . Both Tilak and Annie Besant had played a leading role in bringing about this agreement between the Congress and the League, much against the wishes of many important leaders, including Madan Mohan Malaviya. The pact accepted the principle of separate electorates for the Muslims. Statement 3 is incorrect. The session was presided over by Ambika Charan Majumdar. Source) Chapter 15; page number 335

90. Pairs 1 and 2 are incorrectly matched.

©Jatin Verma All Rights Reserved. https://www.jatinverma.org P a g e | 68

Maulana Azad founded Al Hilal and Mohammad Ali founded Comrade. during the First World War, both the newspapers faced suppression while the leaders such as Ali brothers, Maulana Azad and Hasrat Mohani faced internment. This generated anti-imperialist sentiments among the ‘Young Party’. Pair 3 is correctly matched. Rast Goftar was an Anglo-Gujarati paper operating in Bombay that was started in 1854 by Dadabhai Naoroji and Kharshedji Cama and championed social reform among Parsis in Western India. Source) Chapter 15; page number 336, https://en.wikipedia.org/wiki/Rast_Goftar

91. The British Government established a Disorders Inquiry Committee headed by Lord William Hunter, a Senator of College of Justice of Scotland to inquire into the events in Punjab. It was also termed as the Disorders Inquiry Committee. It was the Simon Commission which arrived in British India in 1928 to study constitutional reform in Britain's largest and most important possession. Source) Chapter 16; page number 366

92. Statement 3 is correct. The Chauri-Chaura Incident (February 5, 1922)—Violence by agitated mob prompted Gandhi to withdraw the Khilafat Non-Cooperation Movement. Statement 1 is incorrect. Khilafat Non-Cooperation Movement brought the urban Muslims into the national movement. Statement 2 is incorrect. The Khilafat Non-Cooperation Movement communalized national politics, to an extent. Although Muslim sentiments were a manifestation of the spread of a wider anti-imperialist feeling, the national leaders failed to raise the religious-political consciousness of the Muslims to a level of secular political consciousness. Source) Chapter 16; page number 378

93. Statement 1 is correct: The Moderates believed in England’s providential mission in India. They believed that political connections with the British are in India’s social-political and cultural interests. The professed loyalty to the British crown. Statement 2 is Incorrect: Moderates insisted on the use of constitutional methods only. It was extremists who used the extra-constitutional method like boycott and passive resistance to achieve their objectives.

94. Option D is the Correct Answer. Dakshina Bharat Hindi Prachar Sabha was established in the year 1918 by Mahatma Gandhi with the sole aim of propagating Hindi in the Southern States. On 30th September 1932, Mahatma Gandhi founded All India Untouchability League, to remove untouchability in the society, which later renamed as Harijan Sevak Sangh The Natal Indian Congress was an organization that aimed to fight discrimination against Indians in South Africa. The Natal Indian Congress was founded by Mahatma Gandhi in 1894.

95. The government formed a Hunter Commission to investigate the Jallianwala Bagh shootings. On October 14, 1919, the Government of India announced the formation of the Disorders Inquiry Committee. The committee was commonly known as Hunter Commission after the name of chairman, Lord William Hunter. It also had Indian members. In the final report submitted in March 1920, the committee

©Jatin Verma All Rights Reserved. https://www.jatinverma.org P a g e | 69

unanimously condemned Dyer’s actions. However, the Hunter Committee did not impose any penal or disciplinary action against General Dyer.

96. In the wake of Khilafat and Home rule agitation, the British government passed the Rowlett Act in March 1919. The act also is known as the” Black Act” or “Black Bill” by the Indians who protested it. Statement 1 is Correct: This act authorized the Government to imprison any person without trial and conviction in a court of law, thus enabling the Government to suspend the right of habeas corpus which had been the foundation of civil liberties in Britain. Statement 2 is Correct: The accused did not have the right to appeal against the court orders. Statement 3 is Incorrect: Regarding the composition of the Court, it laid down that Chief Justice shall nominate three of the High Court judges for the trial of the information, and shall fix a date for the commencement of the trial. It did not explicitly discriminate against the judges of Indian origin.

97. The Non-cooperation movement was formally launched on August 1, 1920. That day was also marked by the death of Lokmanya Tilak, which witnessed strikes and processions to mourn the passing away of this great national leader. It involved the surrender of government titles and honorary positions, boycott of government schools and colleges, law courts and foreign cloth. It was decided to set up national schools and colleges, establish and strengthen the panchayats for settlement of disputes, promotion of hand spinning and weaving, condemnation and renunciation of untouchability, maintenance of communal amity and strict observance of non-violence. It was for the first time, an open extra-constitutional programme of mass mobilization was started by the congress.

98. In line with the government policy contained in Montagu’s statement of August 1917, the government announced further constitutional reforms in July 1918, known as Montagu-Chelmsford or Montford Reform. Based on these, the Government of India Act, 1919 was enacted. Dyarchy – i.e rule of two- Executive councillors and popular ministers was introduced. The governor was to be the executive head in the province.

99. Tolstoy Farm was established by Mahatma Gandhi in South Africa which was made possible through the generosity of his German friend Kallenbach, to house the families of the Satyagraha and give them a way to sustain themselves.

100. The Kakori Conspiracy case was a train robbery that took place between Kakori and, near Lucknow, on 9 August 1925. The resolution for the establishment of a separate homeland for the Muslims of British India passed in the annual session of the All India Muslim League held in Lahore on 22–24 March 1940. , also known as the First World War, was a global war originating in Europe that lasted from 28 July 1914 to 11 November 1918. Chittagong armoury raid, also known as the Chittagong uprising, was an attempt on 18 April 1930 to raid the armoury of police and auxiliary forces from the Chittagong armoury in the Bengal Presidency of British Raj.

©Jatin Verma All Rights Reserved. https://www.jatinverma.org P a g e | 70

101. Lucknow Pact, (December 1916), an agreement made by the Indian National Congress headed by Maratha leader Bal Gangadhar Tilak and the All-India Muslim League led by Muhammad Ali Jinnah; it was adopted by the Congress at its Lucknow session on December 29 and by the league on Dec. 31, 1916.

102. Statement 2 is Correct: In the First World war(1914-1919), Britain allied with France, Russia, USA, Italy and Japan against Germany, Austria, Hungary and Turkey. The nationalist response to British participation in the war was threefold- The Moderates supported the empire in the war as a matter of duty; The extremists, including Tilak, supported the war efforts in the mistaken belief Britain would repay India’s loyalty with gratitude in the form of self-government. The revolutionaries decided to utilize the opportunity to wage a war and liberate the country

103. On 5 February 1922, a clash took place at Chauri Chaura, a small town in the district of , . A police officer had attacked some volunteers picketing a liquor shop. A whole crowd of peasants that had gathered there went to the police chowki (station). The mob set fire to the police chowki with some 22 policemen inside it. Mahatma Gandhi felt that the revolt was veering off-course, and was disappointed that the revolt had lost its non-violent nature. He did not want the movement to degenerate into a contest of violence, with police and angry mobs attacking each other back and forth, victimizing civilians in between. Gandhi appealed to the Indian public for all resistance to end, went on a fast lasting 3 weeks, and called off the non-cooperation movement. 104. The peasants of the Champaran and other areas of North Bihar were growing the Indigo under the tinakathia system. Under the tinakathia system, the peasants were bound to plant 3 out of 20 parts of his land with indigo for his landlord. This means that out of 20 khatas which make an acre, they had to dedicate 3 khatas for indigo plantation. This was the root cause of the trouble. They had to lease this part in the return to the advance at the beginning of each cultivation season. The price was too less and was fixed on the area cultivated rather than the crop produced. They were actually being cheated by the English planters. The planters had agreed to the peasants to relive them from the lease contracts but demanded heavy compensations which they were not able to pay. One local peasant leader Rajkumar Shukla had invited Mahatma Gandhi to visit Champaran. Gandhi Ji arrived in Champaran but was later ordered by the District magistrate of Champaran WB Heycock to leave the district. Gandhi Ji refused and persisted. He decided to commit Satyagraha. He proceeded towards the Champaran. The commissioner of tirhut division ordered Gandhi’s arrest but Government of India cancelled the arrest because it did want to make him a hero.

105. After Gandhi’s arrest (March 1922), there was disintegration, disorganisation and demoralisation among nationalist ranks. Those advocating entry into legislative councils came to be known as the ‘Swarajists’, while the other school of thought led by C. Rajagopalachari, Vallabhbhai Patel, Rajendra Prasad and M.A. Ansari came to be known as the ‘Nochangers’. C.R. Das, Motilal Nehru and Ajmal Khan were prominent leaders of 'Swarajists’ camp and wanted an end to the boycott of legislative councils so that the nationalists could enter them to expose the basic weaknesses of these assemblies and use these councils as an arena of political struggle to arouse popular enthusiasm. Source) Chapter 17; page number 379

©Jatin Verma All Rights Reserved. https://www.jatinverma.org P a g e | 71

106. Statement 1 is incorrect. Gandhi was initially opposed to the Swarajist proposal of council entry. But after his release from prison on health grounds in February 1924, he gradually moved towards a reconciliation with the Swarajists. Statement 2 is incorrect. The Swarajists were allowed to contest elections as a group within the Congress. The Swarajists accepted the Congress programme with only one difference—that they would join legislative councils. The elections to the newly constituted Central Legislative Assembly and to provincial assemblies were to be held in November 1923. statement 3 is correct. The Swarajist released their Manifesto for Elections in October 1923. The Swarajist manifesto took a strong anti-imperialist line. The points put forward were as follows.  The guiding motive of the British in governing India lay in selfish interests of their own country;  The so-called reforms were only a blind to further the said interests under the pretence of granting a responsible government, the real objective being to continue exploitation of the unlimited resources of the country by keeping Indians permanently in a subservient position to Britain;  The Swarajists would present the nationalist demand of self-government in councils;  If this demand was rejected, they would adopt a policy of uniform, continuous and consistent obstruction within the councils to make governance through councils impossible;  Councils would thus be wrecked from within by creating deadlocks on every measure. Source) Chapter 17; page number 381

107. Gandhi was initially opposed to the Swarajist proposal of council entry. But after his release from prison on health grounds in February 1924, he gradually moved towards a reconciliation with the Swarajists due to the following reasons: He felt public opposition to the programme of council entry would be counter-productive. Hence, statement 1 is correct. In the November 1923 elections, the Swarajists had managed to win 42 out of 141 elected seats and a clear majority in the provincial assembly of Central Provinces. In legislatures, in cooperation with the Liberals and the independents like Jinnah and Malaviya, they won a majority. Hence, statement 2 is incorrect. The courageous and uncompromising manner in which the Swarajists functioned convinced him that they would not become just another limb of colonial administration. There was a government crackdown on revolutionary terrorists and the Swarajists towards the end of 1924; this angered Gandhi and he expressed his solidarity with the Swarajists by surrendering to their wishes. Hence, statement 3 is correct. Source) Chapter 17; page number 382

108. The new forces that emerged during the 1920s were inspired by socialists and communists ideas. These ideas also resulted in the rise of a left-wing within the Congress, represented by Jawaharlal Nehru and Subhash Chandra Bose. These younger nationalists— ● were critical of both Swarajists and No-Changers; hence, statement 1 is incorrect. ● advocated a more consistent anti-imperialist line in the form of a slogan for purna swarajya (complete independence); hence, statement 2 is correct.

©Jatin Verma All Rights Reserved. https://www.jatinverma.org P a g e | 72

● were influenced by an awareness, though still vague, of international currents; ● stressed the need to combine nationalism and anti-imperialism with social justice and simultaneously raised the question of internal class oppression by capitalists and landlords. Hence, statement 3 is correct. Source) Chapter 17; page number 387

109. Ramprasad Bismil, Jogesh Chandra Chatterjee and Sachin Sanyal and others formed the Hindustan Republican Association in the United Provinces and started raising funds through dacoities. The most renowned one was the Kakori train robbery in August 1925 that resulted in the arrest of several members of the organization. This organization also established links with a group of young men in the Punjab under the dynamic and brilliant student leader Bhagat Singh. The Punjab group was deeply influenced by socialist ideology. Hence the organization was renamed Hindustan Socialist Republican Association (HSRA). Hence, Bhagat sing was not a founder of the Hindustan Republican Association. The aim of the revolutionaries was complete independence and they had a vision of how the State should be after the achievement of the same. They envisaged a mass struggle of the people and for this purpose they tried to mobilise students workers and peasants. Source) Chapter 17; page number 390

110. The book 'The Philosophy of the Bomb' was written by Bhagwaticharan Vohra. Bhagwati Charan Vohra was an Indian revolutionary, associated with Hindustan Socialist Republican Association. He was an ideologue, organiser, orator and a campaigner. Source) Chapter 17; page number 395

111. The Nehru Report of 28-30 August 1928 was a memorandum outlining a proposed new dominion status constitution for India. Among its important recommendations were: (i) Dominion status on lines of self-governing dominions as the form of government desired by Indians. Hence, statement 1 is incorrect. (ii) Rejection of separate electorates. (iii) Linguistic provinces. (iv) Nineteen fundamental rights including equal rights for women, right to form unions, and universal adult suffrage. (v) Responsible government at the Centre and in provinces; a parliamentary system of government (vi) Adult franchise (vii) An independent judiciary with a supreme court at its head Source) Chapter 18; page number 405, https://nios.ac.in/media/documents/SecSocSciCour/English/Lesson-08.pdf

112. Statement 1 is incorrect. The Congress session in Madras (December 1927) meeting under the presidency of M.A. Ansari decided to boycott the commission “at every stage and in every form”. Statement 2 and 3 are correct. Congress Resolutions at Karachi While disapproving of and dissociating itself from political violence, the Congress admired the ‘bravery’ and ‘sacrifice’ of the three martyrs. The Delhi Pact or Gandhi-Irwin Pact was endorsed.

©Jatin Verma All Rights Reserved. https://www.jatinverma.org P a g e | 73

The goal of purna swaraj was reiterated. Two resolutions were adopted—one on Fundamental Rights and the other on National Economic Programme— which made the session particularly memorable. The Resolution on Fundamental Rights guaranteed— free speech and free press right to form associations right to assemble universal adult franchise equal legal rights irrespective of caste, creed and sex neutrality of state in religious matters free and compulsory primary education protection to culture, language, script of minorities and linguistic groups Source) Chapter 19; page number 426

113. Option B is the Correct Answer: The Second Round Table Conference was held in London from September 7, 1931, to December 1, 1931. The Delhi Pact, also known as the Gandhi-Irwin Pact was signed between the viceroy, representing the British Indian Government, and Gandhi, representing the Indian people, in Delhi on February 14, 1931. Irwin’s Declaration: Before the Simon Commission report came out, the declaration by Lord Irwin was made. It was the combined effort of the Labour Government and a Conservative viceroy. The purpose behind the declaration was to “ restore faith in the ultimate purpose of British policy”. The declaration was made in the form of an official communique in the Indian Gazette on October 31, 1929. Dandi March or Salt Satyagraha: The historic Dandi March, marking the launch of the Civil Disobedience Movement, began on March 12, 1930.

114. Statement 1 is Correct and Statement 2 is Incorrect: Those advocating entry into legislative councils came to be known as the ‘Swarajists’, while the other school 0f thought led by C.Rajagopalachari, Vallabhbhai Patel, Rajendra Prasad and M.A.Ansaricame to be known as the No- changers. The No-changers opposed council entry, advocated concentration on constructive work and continuation of boycott and non-cooperation.

115. Pair 1 is Incorrect: Sarojini Naidu was the second woman (after Annie Besant) and first Indian woman to become congress president. She preceded the Kanpur Session in 1925 Pair 2 is Incorrect: The Indian National Congress, on 19 December 1929, passed the historic Purna Swaraj resolution at its Lahore Session. Pair 3 is Correct: Two resolutions were adopted in 1931, Karachi Congress Session- one on Fundamental Rights and the other on National Economic Programme.

116. Option B is the Correct Answer: B.R. Ambedkar and Tej Bahadur Sapru took part in all the three round table conferences. Mahatma Gandhi took part in the Second Round Table Conference. Prior to the First Round Table Conference, M.K. Gandhi had initiated the Civil Disobedience Movement on behalf of the Indian National Congress. Consequently, since many leaders were in Jail, Congress did not participate in the first conference, but representatives from all other Indian parties participated in it.

©Jatin Verma All Rights Reserved. https://www.jatinverma.org P a g e | 74

117. By their political activities, the Swarajists filled the political vacuum at a time when the national movement was recouping its strength. They demonstrated that the councils could be used creatively by agitating powerful speeches on self-government and civil liberties and outvoting the Government several times, even on matters relating to budgetary grants. They were able to elect Vithalbhai Patel as the speaker of the Central Legislative Assembly in 1925. A noteworthy achievement was the defeat of the Public Safety Bill in 1928 which was aimed at empowering the Government to deport undesirable and subversive foreigners.

118. The school of thought led by Vallabhbhai Patel, Rajendra Prasad, C. Rajagopalachari and M.A. Ansari came to be known as the 'No-changers'. The 'No-changers' opposed council entry, advocated, concentration on constructive work, and continuation of boycott and noncooperation, and quiet preparation for the resumption of the suspended civil disobedience programme. While Those advocating entry into legislative councils came to be known as the Swarajists.

119. Vedaranyam Salt Satyagraha was a march started by C. Rajagopalachari, inspired by the Dandi March.C. Rajagopalachari, a close associate of Gandhi, led the march which had close to 150 volunteers, most of who belonged to the Indian National Congress. It began at Trichinopoly (now Tiruchirappalli) on 13 April 1930 and proceeded for about 150 miles towards the east before reaching Vedaranyam, a small coastal town in the then Tanjore District (Madras Presidency).

120. Option C is the Correct Answer: The Motilal Nehru Report 1928 was a report by a committee headed by Pt. Motilal Nehru. This committee was created when Lord Birkenhead, Secretary of State of India asked the Indian leaders to draft a constitution for the country. The report, which demanded a Dominion Status for India was considered by the Congress. Jawaharlal Nehru was secretary of the committee and Ali Imam, Tej Bahadur Sapru, M.S. Aney, Mangal Singh, Shuaib Qureshi, and G. R. were its members.

121. All the statements given above are Correct: To carry forward the mandate given by the Lahore Congress, Gandhi presented eleven demands to the government and gave an ultimatum of January 31, 1930, to accept or reject these demands. The demands were as follows: Issues of general interests Reduce expenditure on Army and Civil services by 50 per cent. Prohibition of intoxicants and liquor. Carry out reforms in the Criminal investigations Department. Change Arms act allowing popular control of the issue of firearms licences. Release political prisoners. Accept Postal Reservation Bill. Specific Bourgeois Demands Reduce Rupee-sterling exchange ratio. Introduce textile production. Reserve coastal shipping for Indians. Specific Pesant Demands Reduce land revenues by 50 per cent. Abolish salt tax.

©Jatin Verma All Rights Reserved. https://www.jatinverma.org P a g e | 75

122. There were certain aspects in which the CDM differed from the NCM Statement 1 is not correct: The stand objective in CDM was complete independence and not a vaguely worded Swaraj. The methods involved violation of the law from the very beginning and not just non-cooperation with foreign rule. There was a decline in the forms of protests involving the Intelligentsia, such as lawyers giving up the practice, students giving up government schools to join national schools and colleges. Statement 2 is not correct: Muslim participation was nowhere near that in the Non-cooperation Movement level.

123. Statement 1 is Incorrect and statement 2 is Correct: The nationalist response to British participation in the war was threefold- The Moderates supported the empire in the war as a matter of duty The Extremists, including Tilak, supported the war efforts in the mistaken belief that Britain would repay India’s loyalty with gratitude in the form of self-government. The revolutionaries decided to utilize the opportunity to wage a war and liberate the country.

124. Pair 1 is Incorrect: In Punjab, Ghulam Hussain and others published Inquilab. Pair 2 is Correct: In Bombay, S.A.Dange published a pamphlet, Gandhi and Lenin and the first socialist weekly, The Socialist. Pair 3 is Correct: In Madras, M.Singaravelu founded Labour Kisan Gazette.

125. The phase of the Civil Disobedience movement after the second Round Table Conference could not be sustained for long because (i) Gandhi and other leaders had no time to build up the tempo; hence, statement 1 is incorrect. (ii) The masses were not prepared. Hence, statement 2 is correct. (iii) In April 1934, Gandhi decided to withdraw the civil disobedience movement. Though people had been cowed down by superior force, they had not lost political faith in the Congress—they had won freedom in their hearts. Hence, statement 3 is incorrect. Source) Chapter 20; page number 434 126. The Poona Pact was the agreement between Mahatma Gandhi and Dr. Br Ambedkar reached on 25 September 1932. Chakravarti Rajagopalachari was not associated with the Poona Pact. This Poona Pact was signed by Madan Mohan Malviya, Ambedkar and some other leaders as a means to end the fast that Gandhi was undertaking in jail as a protest against the decision by British Prime Minister Ramsay Macdonald to give separate electorates to Dalits for the election of members of provincial legislative assemblies in British India. They finally agreed upon 148 seats. Source) Chapter 20; page number 438

127. The Government of India Act, 1935 act divided the subjects to be administered into reserved and transferred subjects. Reserved subjects—foreign affairs, defence, tribal areas and ecclesiastical affairs— were to be exclusively administered by the governor-general on the advice of executive councillors. Source) Chapter 20; page number 452

©Jatin Verma All Rights Reserved. https://www.jatinverma.org P a g e | 76

128. Statement 1 is incorrect. Subhash Chandra Bose was elected as the President of the Congress at Haripur (1938) session and Tripuri session by defeating Gandhi’s favorite candidate Pattabhi Sitaramayya (1939). He could not make any compromise with Gandhi and thus resigned his President-ship. Statement 2 is incorrect. After resigning from the Congress, he organized the famous Forward Block. Forward Bloc (at Makur, ) was a new party within the Congress. Source) Chapter 22; page number 471

129. Mahatma Gandhi did not initiate a movement immediately after the beginning of World War-II because he and some other dominant leaders felt that the cause of Allies was just. Jawaharlal Nehru considered the Allied powers as imperialists and his philosophy and political leant towards the idea of an early struggle but that would have undermined the fight against Facism. Instead of an immediate mass struggle, Gandhi advocated toning up the Congress organisation, carrying on political work among the masses, and negotiating till all possibilities of a negotiated settlement were exhausted. Source) Chapter 22; page number 491

130. The august Offer turned down the demand of the Indian National Congress to set up a national Government at the centre but proposed the following: 1. After the war, a representative “Constitution Making Body” shall be appointed immediately after the war. 2. The number of the Indians in the Viceroy’s Executive council will be increased. 3. A war advisory Council would be set up. Source) Chapter 22; page number 493

131. Both the statements are correct. The aims of launching Individual Satyagraha were- (i) to express people's feeling that they were not interested in the war and they had made no distinction between Nazism and the double autocracy that ruled India; ii) to show the nationalist patience was not due to weakness; iii) to give another opportunity to the Government to Congress' demands peacefully. Gandhi decided to initiate a limited Satyagraha on an individual basis by a fewselected individuals in every locality. The demand of the Satyagrahi would be the freedom of speech against the war through an anti-war declaration. (2nd World War). Source) Chapter 22; page number 494

132. Statement 1 is incorrect. The Crips mission did not mention anything about the creation of Pakistan. Statement 2 is correct. It proposed a single Indian union. The Muslim league criticized this measure. Statement 3 is correct. It offered a dominion status. It offered to create a constitutional assembly only after the Second World War was over. Cripps Mission was sent by the British Government in 1942. It came up with constitutional proposals to seek Indian support for ongoing Second World War which in British was involved. Source) Chapter 22; page number 496

©Jatin Verma All Rights Reserved. https://www.jatinverma.org P a g e | 77

133. Statement 1 is Correct: The Government of India Act was passed by the British Parliament in August 1935. It provided for the establishment of an All-India Federation consisting of all British Indian provinces, all chief commissioner’s provinces and the Indian states (princely states) as units. Statement 2 is Incorrect: The federal subjects were divided into twofold categories of Reserved and Transferred subjects. The Reserved list comprised of subjects such as administration of defence, external affairs, ecclesiastical affairs and matters related to tribal areas. These subjects were administered by the Governor-General in his discretion with the help of three councillors appointed by him. But Executive councillors were not responsible to the Central legislature. The administration of the transferred subjects was to be done by Governor-General on the advice of the Council of Ministers, whose number could not exceed 10.

134. Option D is the Correct Answer: Main incidents during the Viceroyship of Lord Willingdon (1931- 36) are: Second and Third Round Table conferences. Announcement of Communal award (1932) by Ramsay Mac Donald. Poona pact between Gandhi and Ambedkar (1932). Enactment of the Government of India Act 1935. Foundation of Socialist Party by Acharya Narendra Dev and Jai Prakash Narayan (1934) and the All India Kisan Sabha (1936). Burma separated from India (1935) Launch of Individual civil disobedience (1933).

135. Statement 2 is Incorrect: The Cripps Mission was sent by the British government to India in March 1942 to obtain Indian cooperation for the British war efforts in the 2nd World War. It was headed by Sir Richard , a labour minister in Winston Churchill’s coalition government in Britain. Main proposals of the Cripps mission were, An Indian Union with a dominion status would be set up under the British Government.

136. Option C is the Correct Answer The Indian National Congress met at Haripura from 19 to 22 February 1938, under the presidency of Subhas Chandra Bose; he was elected President of the Haripura Congress Session in 1938. The Viceroy of India, Lord Linlithgow, issued a statement from Simla on 8 August 1940 in the response of when congress formally asks England to affirm its adherence to the goal of Independence for India which is popularly known August Offer. World War II, also known as the Second World War, was a global war that lasted from 1939 to 1945. World War II officially began on September 1, 1939. Cripps Mission was sent by the British Government in March 1942 to India with key objective to secure Indian cooperation and support for British War Efforts. Headed by Sir Stafford Cripps, this mission sought to negotiate an agreement with Indian leaders.

137. The aims of launching individual satyagraha were- To show that nationalist patience was not due to weakness To express people’s feeling that they were not interested in the war and that they made no distinction between Nazism and the double autocracy that ruled India. Hence Statement 1 is Correct.

©Jatin Verma All Rights Reserved. https://www.jatinverma.org P a g e | 78

To give another opportunity to the government to accept Congress demands peacefully. Statement 2 is Incorrect: was the first to offer the satyagraha and Nehru, the second.

138. Pair 1 is Correct: Bombay Session of the INC in 1934 presided by Rajendra Prasad Pair 2 is Incorrect: Faizpur Session of the INC in 1936 presided by Jawaharlal Nehru. It was here, for the first time that Congress held its Annual Session in a backward rural setting. Pair 3 is Incorrect: Subhash Chandra Bose was re-elected the President of INC at the Tripuri Session in 1939 by defeating Pattabhai Sitaramaiyya.

139. Option D is the Correct Answer: Congress ministries were formed in Bombay, Madras, Central provinces, Orissa, United Provinces, Bihar and later in NWFP and Assam also. 140. Statement 1 is correct. Some Sikh leaders were demanding a separate state if the country was partitioned. The principle of communal self –determination would claim separation of Hindu majority Western Bengal and Sikh and Hindu dominated Ambala and Jullunder divisions of Punjab. Statement 2 is correct. Pakistan so formed would include a large non-Muslim population; 38% in the North West and 48% in the North East. Statement 3 is correct. Considering the vastness of the country and scattered economic resources and infrastructures, there was an obvious implication of economic and administrative problems that might occur due to the partition. Statement 4 is correct. The deep-seated regional ties would be disturbed if Bengal and Punjab were partitioned. Source) Chapter 24; page number 530

141. Statement 1 is incorrect. At a meeting of the Congress Working Committee in Wardha passed the Quit India Resolution on 14th July 1942 which was later endorsed and passed on 8th August at the Bombay session of the Congress. Statement 2 is correct. The Quit India Resolution was ratified at the Congress session at Gowalia Tank, Bombay. The meeting also resolved to 1. demand an immediate end to British rule in India. 2. declare commitment of free India to defend itself against all types of Fascism and imperialism. 3. form a provisional Government of India after British withdrawal. 4. sanction a civil disobedience movement against British rule. 5. Gandhi was named the leader of the struggle Source) Chapter 23; page number 502

142. Gandhi's special instructions were spelt out at the Gowalia Tank meeting but not actually issued. They were directed at various sections of society. Government servants: Do not resign but declare your allegiance to the Congress. Hence, statement 2 is incorrect. Soldiers: Do not leave the Army but do not fire on compatriots. Hence, statement 2 is incorrect. Students: If confident, leave studies. Peasants: If zamindars are anti-government, pay mutually agreed rent, and if zamindars are pro- government, do not pay rent. Princes: Support the masses and accept the sovereignty of your people. Hence, statement 3 is correct.

©Jatin Verma All Rights Reserved. https://www.jatinverma.org P a g e | 79

Princely states' people: Support the ruler only if he is anti-government and declare yourselves to be a part of the Indian nation. Source) Chapter 23; page number 503

143. All the statements are correct. During the Quit India movement, the students responded by going on strike in schools and colleges, participating in processions, writing and distributing illegal news sheets (patrikas) and acting as couriers for underground networks. Workers went on strike in , Bombay, Jamshedpur, Ahmednagar and Poona. Source) Chapter 23

144. Statement 1 is correct. The Communists did not join the movement; in the wake of Russia (where the communists were in power) being attacked by Nazi Germany, the communists began to support the British war against Germany and the ‘Imperialist War’ became the ‘People’s War’. Statement 2 is correct. Government officials, especially those belonging to lower levels in police and administration, participated resulting in erosion of government loyalty. Statement 3 is incorrect. Peasants of all strata were at the heart of the movement. Even some zamindars participated. These peasants concentrated their offensive on symbols of authority and there was complete absence of anti-zamindar violence. Source) Chapter 23; page number 506

145. The Muslim League opposed the movement, fearing that if the British left India at that time, the minorities would be oppressed by the Hindus. The Hindu Mahasabha boycotted the movement. The Princely states showed a low-key response. Hence, the correct answer is B. Source) Chapter 23; page number 506

146. All were associated with the Indian National Army. Shah Nawaz commanded the INA batallion that accompanied the Japanese Army in the Imphal Campaign. Rashbehari Bose assisted Subhash Chandra Bose in Singapore. Prem Kumar Sehgal was one of the INAprisoners of war. Gurbaksh Singh Dhillon was an officer in the Indian National Army who was charged with "waging war against His Majesty the King Emperor". Along with Shah Nawaz Khan and Prem Kumar Sahgal, he was tried at the end of World War II in the INA trials that began on 5 November 1945 at Red Fort. Source) Chapter 23 and chapter 24; page number 515

147. Statement 1 is correct. After Cripps's departure, Gandhi framed a resolution calling for British withdrawal and a non-violent non-cooperation movement against any Japanese invasion. The CWC meeting at Wardha (July 14, 1942) accepted the idea of a struggle. The failure of the Cripps Mission to solve the constitutional deadlock exposed Britain's unchanged attitude on constitutional advance and made it clear that any more silence would be tantamount to accepting the British right to decide the fate of Indians without consulting them. Statement 2 is correct. Various underground activities were one of the dominant features of the Quit India movement. Usha Sharma started an underground radio in Bombay.

©Jatin Verma All Rights Reserved. https://www.jatinverma.org P a g e | 80

Statement 3 is correct. There were no communal clashes during the Quit India movement. Muslims helped by giving shelter to underground Activists. Source) Chapter 23; page number 504

148. All of the given statements was the fundamental cause of the 1943 famine. 1943 Bengal famine, which is estimated to have caused over three million deaths, resulted not from drought as is widely thought but from the British government's policy failures. The fundamental causes of the famine were as follows: The need to feed a vast Army diverted foodstuffs. Rice imports from Burma and South East Asia had been stopped. The famine got aggravated by gross mismanagement and deliberate profiteering; rationing methods were belated and were confined to big cities.

149. The objectives of Rajagopalachari Formula were to bridges between the Muslims League and Indian National Congress arises due to the difference of opinion on two-nation theory and independence of India from British. C. Rajagopalachari was the veteran Congress leader, made a formula for the Indian National Congress and Muslim League cooperation to write off the political deadlock. The formula was supported by M.K Gandhi which was in reality, silent acceptance of the League’s demand for Pakistan. The main points in C.Rajagopalachari’s Formula were Muslim League to support the demand for independence of Congress. Hence statement 2 is Correct. League to cooperate with Congress in forming a provisional government at the centre. Hence statement 3 is Correct. At the end of the war, a commission would be appointed to demarcate the districts having a Muslim population in the absolute majority and in elections to be conducted in those areas (including the non- Muslims) on basis of adult suffrage whether or not to form a separate sovereign state. All parties would be allowed to express their stand on the partition and their views before the voting or election. In case of acceptance of partition, agreement to be made jointly for safeguarding defence, commerce, communications, etc. The above terms to be operative only if England transferred full powers to India. Hence statement 4 is Correct.

150. Statement 1 is Correct: In July 1942, the Congress Working Committee met at Wardha and resolved that it would authorise Gandhi to take charge of the non-violent mass movement. The resolution generally referred to as ‘the Quit India Resolution’. Statement2 is Incorrect: The Quit India Resolution proposed by Jawaharlal Nehru and seconded by Sardar Patel.

151. On August 8, 1942, the Quit India Resolution was passed at the Bombay session of the All India Congress Committee (AICC). C. Rajagopalachari, the veteran Congress leader prepared a formula for Congress-League cooperation in 1944. Desai being the leader of the Congress in the Central Assembly and a friend of Liaquat Ali (Leader of Muslim League), met him in January 1945 gave him proposals for the formation of Interim Government

©Jatin Verma All Rights Reserved. https://www.jatinverma.org P a g e | 81

at the centre. After Desai’s declaration, Liaquat Ali published the list of an agreement, which came to be known as Desai-Liaqat Pact. Lord Wavell became the Viceroy of India in 1943 succeeding Lord Linlithgow. In June 1945, he announced his proposals to break the constitutional deadlock in India. This was called the Wavell Plan.

152. Statement 1 is Correct: Parallel government was established under Chittu Pandey in Balia(in August 1942 for a week), the town of United Province. Statement 2 is Correct: Jatiya Sarkar was an independent parallel government established in Tamluk, Bengal(From December 1942 to September 1944). The Jatiya Sarkar undertook cyclone relief work, sanctioned grants to schools, supplied paddy from the rich to the poor, organised Visyut Vahinis, etc. Statement 3 is Incorrect: Leaders like Yashwant Chavan, Nana Patil established Prati Sarkar in Satara(mid-1943-1945). Village libraries and Nyaydan Mandals were organised, prohibition campaigns were carried on and ‘Gandhi marriages’ were organised.

153. The British Government sent a delegation to India under Sir Stafford Cripps, to negotiate with the Indian National Congress a deal to obtain total cooperation during the war, in return of progressive devolution and distribution of power from the crown and the viceroy to an elected Indian legislature. The talks failed, as they did not address the key demand of a time table of self-government and of the definition of the powers to be relinquished, essentially making an offer of limited dominion status that was wholly unacceptable to the Indian Movement.

154. All of the above personalities were associated with the Quit India Movement Bhulabhai Desai was the arch-Moderates, who were in the favour of recognising the rights of Muslim majority provinces to secede through plebiscites after independence had been gained. Satish Samanta, a Congress leader and the first Sarbadhinayak of the Tamluk Jatiya Sarkar, helped in establishing a rebel ‘national government’ in Tamluk subdivision of Midnapur. Usha Mehta actively supported the movement and was an important member of a small group which ran the Congress radio. Subhash Chandra Bose joined the Indian National Army in 1943. One of his most famous declarations wa’Tum mujhe khoon do mai tumhe Azadi doonga’

155. Usha Mehta (March 25, 1920-August 11, 2000) was a renowned Gandhian and freedom fighter of India. She is also remembered for operationalizing the Congress radio, also called the Secret Congress Radio, an underground radio station, which functioned for few months during the Quit India Movement of 1942

156. Statement D is the Correct: On August 8, 1942, the Quit India Resolution was passed in the Bombay Session of the All India Congress Committee and here Gandhi made a call to participate people in a non- violent way. It’s worth nothing that the communists had opposed this movement. Labour Unions under Communist influence had apparently decided against participation in the movement, there were large scale strikes in mills at Kanpur, Jamshedpur, and Ahmedabad. There was the indifference of the Labour Class.

©Jatin Verma All Rights Reserved. https://www.jatinverma.org P a g e | 82

157. The parties established by Dr B.R Ambedkar were- All India Scheduled Castes Federation and the Independent Labour Party.

158. The Congress ministers resigned in October and November 1939, in protest against Viceroy Lord Linlithgow action of declaring India to be a belligerent in the Second World War without consulting the Indian people.

159. In March 1942, a mission headed by Stafford Cripps was sent to India with constitutional proposals to seek Indian support for the war by British PM, Winston Churchill. Nehru and Maulana Azad were the official negotiators of the Congress.

160. Statements 2, 3 and 4 are correct. After the second world war, there a change in the British Government's Attitude, because 1. The end of the war resulted in a change in the balance of global power—the UK was no more a big power while the USA and USSR emerged as superpowers, both of which favoured freedom for India. Hence, statement 1 is incorrect. 2. The new Labour government was more sympathetic to Indian demands. 3. Throughout Europe, there was a wave of socialist radical governments. 4. British soldiers were weary and tired and the British economy lay shattered. (By 1945 the British government in London owed India £1.2 billion and was being drained by the US Lend-Lease agreement, which was finally paid off only in 2006.) 5. There was an anti-imperialist wave in South-East Asia—in Vietnam and Indonesia—resisting efforts to replant French and Dutch rule there. 6. Officials feared another Congress revolt, a revival of the 1942 situation but much more dangerous because of a likely combination of attacks on communications, agrarian revolts, labour trouble, army disaffection joined by government officials and the police in the presence of INA men with some military experience. 7. Elections were inevitable once the war ended since the last elections had been held in 1934 for the Centre and in 1937 for the provinces. The British would have had to retreat; the Labour government only quickened the process somewhat. Source) Chapter 24; page number 519

161. Statements 1 and 2 are correct. The Congress was willing to accept dominion status despite its being against the Lahore Congress (1929) spirit because (i) it would ensure a peaceful and quick transfer of power; (ii) it was more important for the Indian National Congress to assume the authority to check the explosive situation; hence, statement 3 is incorrect. (iii) it would allow for some much-needed continuity in the bureaucracy and the army. Source) Chapter 25; page number 554

162. Both the statements are incorrect. The Act provided for the creation of two independent dominions of India and Pakistan with effect from August 15, 1947. Each dominion was to have a governor-general to be responsible for the effective operation of the Act.

©Jatin Verma All Rights Reserved. https://www.jatinverma.org P a g e | 83

The constituent assembly of each new dominion was to exercise the powers of the legislature of that dominion, and the existing Central Legislative Assembly and the Council of States were to be automatically dissolved. For the transitional period, i.e., till a new constitution was adopted by each dominion, the governments of the two dominions were to be carried on in accordance with the Government of India Act, 1935. Source) Chapter 25; page number 555

163. The Pitt's India Act gave the British government a large measure of control over the Company's affairs. The Company became a subordinate department of the State. The Company's territories in India were termed 'British possessions'. Source) Chapter 26; page number 565

164. Statement 1 is incorrect. Through the Charter Act of 1813, the Company's monopoly over trade in India ended, but the Company retained the trade with China and the trade-in tea. Statement 2 is incorrect. The Company's shareholders were given a 10.5 percent dividend on the revenue of India. Statement 3 is incorrect. The powers of the Board of Control were further enlarged. The constitutional position of the British territories in India was defined explicitly for the first time. The Company was to retain the possession of territories and the revenue for 20 years more, without prejudice to the sovereignty of the Crown. Source) Chapter 26; page number 566

165. All the statements are correct. The Indian Councils Act, 1909 is known as the Morley-Minto Reforms, the Act made the first attempt to bring in a representative and popular element in the governance of the country. The strength of the Imperial Legislative Council was increased. With regard to the central government, an Indian member was taken for the first time in the Executive Council of the Governor-General (Satyendra Prasad Sinha was the first Indian to join the Governor- General’s—or Viceroy’s—Executive Council, as law member.) The members of the Provincial Executive Council were increased. Besides separate electorates for the Muslims, representation in excess of their population strength was accorded to the Muslims. Also, the income qualification for Muslim voters was kept lower than that for Hindus. Source) Chapter 26; page number 571 166. The freedom-with-partition formula was coming to be widely accepted well before Mountbatten arrived in India. It is also known as the 3rd June Plan. The important points of the plan were as follows: Punjab and Bengal Legislative Assemblies would meet in two groups, Hindus and Muslims, to vote for partition. If a simple majority of either group voted for partition, then these provinces would be partitioned. In the case of partition, two dominions and two constituent assemblies would be created. Sindh would take its own decision. Hence, statement 3 is correct. Referendums in NWFP and Sylhet district of Bengal would decide the fate of these areas. Hence, statement 2 is incorrect.

©Jatin Verma All Rights Reserved. https://www.jatinverma.org P a g e | 84

The League’s demand was conceded to the extent that Pakistan would be created and the Congress’ position on unity was taken into account to make Pakistan as small as possible. Mountbatten’s formula was to divide India but retain maximum unity. Hence, statement 1 is correct. Source) Chapter 25; page number 553

167. Through the Charter Act of 1833, all restrictions on European immigration and the acquisition of property in India were lifted. Thus, the way was paved for the wholesale European colonization of India. Source) chapter 26; page number 566

168. Provisions of 1773 Regulating Act: This act permitted the company to retain its territorial possessions in India but sought to regulate the activities and functioning of the company. It did not take over power completely, hence called ‘regulating’. The act provided for the appointment of a Governor-General along with four Councillors in the Presidency of Fort William (Calcutta), jointly called the Governor-General in Council. As per this, Warren Hastings was appointed as the Governor-General of the Presidency of Fort William. The Governors in Councils at Madras and Bombay were brought under the control of Bengal, especially in matters of foreign policy. Now, they could not wage war against Indian states without Bengal’s approval. The company directors were elected for a period of five years and one-fourth of them were to retire every year. Also, they could not be re-elected. The company directors were directed to make public all correspondence on revenue, civil and military matters with Indian authorities before the British authorities. A Supreme Court of Judicature was established at Calcutta with Sir Elijah Impey as the first Chief Justice. Judges were to come from England. It had civil and criminal jurisdiction over the British subjects and not Indian natives.

169. The Vernacular Press Act introduced by Lord Lytton in 1878.

170. Statement 1 and 3 is Correct: The Government of India Act, 1919 divided the provincial subjects into two parts—transferred and reserved. The transferred subjects were to be administered by the governor with the aid of ministers responsible to the Legislative Council. The reserved subjects, on the other hand, were to be administered by the governor and his executive council without being responsible to the Legislative Council. This dual scheme of governance was known as ‘dyarchy’—a term derived from the Greek word di-arche which means a double rule. Statement 2 is Incorrect: It extended the principle of communal representation by providing separate electorates for Sikhs, Indian Christians, Anglo-Indians and Europeans.

171. Lord Cornwallis is usually known as the Father of civil services in India. He had introduced the Covenanted Civil Services and the Uncovenanted Civil Services. The Covenanted Civil Services was created out of the Law of the Company. It was the higher civil services and comprised, almost exclusively, of Europeans who were paid very high salaries. The Uncovenanted Civil Services were the lower civil services and comprised mostly of native-born Indians, and to some extent, Europeans too. They were not paid as high as the Covenanted Civil Services.

©Jatin Verma All Rights Reserved. https://www.jatinverma.org P a g e | 85

172. Option A is the Correct Answer. 1. Lord Dalhousie (1848-1856) 2. Lord Mayo (1869-1872) 3. Lord Ripon (1880-1884) 4. Lord Dufferin (1884-1888) 5. Lord Irwin (1926-1931) 173. Statement 1 is Incorrect: Lord Cornwallis organised a regular police force to maintain law and order by going back to and modernising the old Indian system of thanas (circles) in a district under a daroga (an Indian) and a superintendent of police (SP) at the head of a district. Statement 2 is Correct: A Supreme Court of Judicature was established at Calcutta with Sir Elijah Impey as the first Chief Justice by the Regulating Act, 1773. Statement 3 is Correct: The Indian Penal Code (IPC) is the official criminal code of India. It is a comprehensive code intended to cover all substantive aspects of criminal law. The code was drafted in 1860 on the recommendations of first law commission of India established in 1834 under the Charter Act of 1833 under the Chairmanship of Lord Thomas Babington Macaulay.

174. All of the pairs given above are correct. Lord Hardinge II (1910-1916): Creation of Bengal presidency in 1911. Transfer of capital from Calcutta to Delhi (1911). Establishment of Hindu Mahasabha (1915) by Madan Mohan Malviya. Coronation Durbar of King held in Delhi (1911) known as ‘Delhi Durbar’. Lord Irwin (1926-1931): Visit of Simon Commission to India (1928) and the boycott of the commission by the Indians. Appointment of Harcourt Butler Indian States commission (1927). Dandi March (March 12, 1930) by Gandhi. Lord Willingdon (1931-1936): Second Round Table Conference (1931) Announcement of Communal Award (1932) Fast unto death by Gandhi in Yerwada jail, broken after the Poona Pact (1932). Third Round Table Conference (1932).

175. Statement 1 is correct. The Anglicists school of thoughts argued that government spending on education should be exclusively for modern studies. Statement 2 is correct. The Orientalists said while western sciences and literature should be taught to prepare students to take up jobs, emphasis should be placed on the expansion of traditional Indian learning Statement 3 is correct. Even the Anglicists school of thoughts were divided over the question of medium of instruction-one faction was for the English language as the medium, while the Orientalists school of thoughts faction was for Indian languages (vernaculars) for the education purpose. Source) Chapter 30

©Jatin Verma All Rights Reserved. https://www.jatinverma.org P a g e | 86

176. James Thomson was the lieutenant-governor of NW Provinces (1843-53). He developed a comprehensive scheme of village education through the medium of vernacular languages. In these village schools, useful subjects such as mensuration and agriculture sciences were taught. The purpose was to train personnel for the newly set up Revenue and Public Works Department. Source) Chapter 29; page number 635

177. Both the statements are correct. The Press Act of 1835 or Metcalfe Act, Metcalfe (governor-general- 1835-36) repealed the obnoxious 1823 ordinance (According to these regulations, starting or using a press without licence was a penal offence)The new Press Act (1835) required a printer/publisher to give a precise account of premises of a publication and cease functioning, if required by a similar declaration. The Press Act of 1835 turned as the "liberator of the Indian press", The result of a liberal press policy was a rapid growth of newspapers. Source) Chapter 30

178. The 'Cunningham Circular' was implemented in 1930 by the British To stem the students' participation in the Civil Disobedience Movement. This ruling forbade students from participating in political activities and so, evoked strong response. Source) Chapter 29 179. Statement 1 is incorrect. The Hartog Committee recommended that emphasis should be given to primary education but there need be no hasty expansion or compulsion in education. Statement 2 is correct. An increase in a number of schools and colleges had led to the deterioration of education standards. A Hartog Committee was set up to report on the development of education and suggest measure from preventing deterioration of education standards. Source) Chapter 29

180. Press Act of 1835 or Metcalfe Act, Metcalfe (governor-general-1835-36) repealed the obnoxious 1823 ordinance and earned the epithet, "liberator of the Indian press". The new Press Act (1835) required a printer/publisher to give a precise account of premises of a publication and cease functioning if required by a similar declaration. The result of a liberal press policy was a rapid growth of newspapers. Source) Chapter 30

181. Fort William College was set up by Lord Richard Colley Wellesley(Arthur Wellesley was different ) in 1800 for the training of civil servants of the Company in languages and customs of Indians (closed in 1802). The Sanskrit College was established by Jonathan Duncan, the resident, at Benaras in 1791 for the study of Hindu law and philosophy. John Elliot Drinkwater founded the Bethune School at Calcutta in 1849. It was the first fruit of a powerful movement for the education of women which arose in the 1840s and 1850s. The Calcutta Madrasah was established by Warren Hastings in 1781 for the study of Muslim law and related subjects. The pair 1 is incorrectly matched. Source) Chapter 29

©Jatin Verma All Rights Reserved. https://www.jatinverma.org P a g e | 87

182. Raja Ram Mohan Roy (1772-1833) was the founder and editor of two vernacular weekly newspapers viz. Sambad Kaumudi (in Bengali) and Mir’at’l-Akhbar (in the Persian) and as the founding member of the English Newspaper “Bengal Herald’. Source) Chapter 30

183. Pair 1 and 3 is Correct: The Calcutta Madrasah was established by Warren Hastings in 1781 for the study of Muslim Law and related subjects. The Sanskrit College was established by Jonathan Duncan in 1791 for Hindu law and philosophy. Pair 2 is Incorrect: Fort William College was set up by Richard Wellesly in 1800 for the training of civil servants of the Company in languages and customs of Indians

184. In 1854, Charles Wood prepared a despatch on an educational system for India. Considered the “Magna Carta of English Education in India”, this document was the first comprehensive plan for the spread of education in India. It asked the Government of India to assume responsibility for the education of the masses. It systematized the hierarchy from vernacular primary schools in villages at the bottom, followed by Anglo-Vernacular High Schools and an affiliated college at the district level, and affiliating universities in the presidency towns of Calcutta, Bombay and Madras. Recommended English as the medium o instruction for higher studies and vernaculars at school level. Introduced secular education in government institutions. Recommended a system of grants-in-aid to encourage private enterprise.

185. Hunter Education Commission, 1882-83 Indian Universities Act, 1904 Saddler University commission, 1917-19 Hartog Committee, 1929

186. Raleigh Commission was appointed under the presidency of Sir Thomas Raleigh on 27 January 1902 to inquire into the condition and prospects of universities in India and to recommend proposals for improving their constitution and working. Evidently, the Commission was precluded from reporting on primary or secondary education. As a result of the report of the recommendations of the Commission the Indian Universities Act was passed in 1904. 187. The Vernacular Press Act (VPA) was designed to better control the vernacular press and effectively punish and repress seditious writing. The provisions of the Act are as follows: Statement 1 is Correct: The District Magistrate was empowered to call upon the printer and publisher of any vernacular newspaper to enter into a bond with the government undertaking not to cause disaffection against the government or antipathy between persons of different religions, caste, race through published material: the printer and publisher could also be required to deposit security which could be forfeited if the regulation were contravened, and press equipment could be seized if the office re-occurred. Statement 2 is Incorrect: The magistrate action was final and no appeal could be made in a court of law. Statement 3 is Correct: Under VPA, proceedings were instituted against Som Prakash, Bharat Mihir, Dacca Prakash and Samachar.

©Jatin Verma All Rights Reserved. https://www.jatinverma.org P a g e | 88

188. James Augustus Hickey in 1780 started The Bengal Gazette or Calcutta General Advertiser, the first newspaper in India, which was seized in 1872 because of its outspoken criticism of the Government.

189. Somaprakash-Dwarkanath Visyabhushan Rast Goftar- Dadabhai Naoroji Sambad Kaumudi- Raja Rammohan Roy

190. Only Statement 1 is correct and Statement 2 and 3 is Incorrect Wardha Scheme of a Basic Education: The Congress had organised a National Conference on Education in October 1937 in Wardh. In the light of the resolutions passed there. Zakir Hussain Committee formulated a detailed national scheme for basic education. The main principle behind this scheme was ‘learning through activity’. It was based on Gandhi’s ideas published in a series of articles in the weekly Harijan. The scheme had the following provisions: Inclusion of a basic handicraft in the syllabus. First seven years of schooling to be an integral part of a free and compulsory nationwide education system. Teaching to be in Hindi from class II to VII and in English only after class VIII. Ways to be devised to establish contact with the community around schools through service. A suitable technique to be devised with a view to implementing the main idea of basic education- educating the child through the medium of the productive activity of a suitable handicraft.

191.  Mughal Emperor Akbar was very fond of painting and during his reign, the painting was organized as an Imperial establishment or karkhana called Tasvir Khana. 192.  The Fathepur Sikri was founded as the capital of the in 1571 by Emperor Akbar, There are many structures like mosques, palaces, tombs, etc built in the complex of Fatehpur Sikri. The name of some of them are −  Buland Darwaza  Jama Masjid  Ibadat Khana  Jamat Khana  Tomb of Salim Chishti  Diwan-i-Aam  Diwan-i-Khas  Jodha Bai Palace  Panch Mahal  Birbal’s House  Anup Talao  Hujra-i-Anup Talao  Naubat Khana  Pachisi Court  Hiran Minar  The Moti Masjid is a white marble mosque inside the Red Fort complex in Delhi.

©Jatin Verma All Rights Reserved. https://www.jatinverma.org P a g e | 89

193. Akbar’s patronage attracted the best painters to his court. The ablest and the most numerous among them were Hindu painters. They were employed to paint the walls of Akbar’s capital at Fatehpur Sikri and also to produce albums. The important Hindu painters in the court of Akbar were: Daswanth, Baswan, Mukund, Jagan, Kesu lal etc. Aqa Riza, Abul Hasan, Mansur, Bishan Das, Manohar, Goverdhan, Balchand, Daulat, Mukhlis, Bhim and Inayat were the famous painters in the court of Jahangir.

194. Under Jahangir, painting acquired greater charm, refinement and dignity. He had a great fascination for nature and took delight in the portraiture of birds, animals and flowers. Some important manuscripts illustrated during his period are, an animal fable book called Ayar-i-Danish, the leaves of which are in the Cowasji Jahangir collection, Bombay and the Chester Beatty Library, Dublin, and the Anwar-i- sunavli, another fable book in the British Museum, London, both executed between 1603-10, some miniatures in the Gulistan and a Diwan of Hafiz both in the British Museum.

195. Both statement 1 and 2 is Incorrect: This tomb of Itmad ud Daulah was built just outside the wall of Agra, made with marbles. The mausoleum is placed on a large red sandstone platform, which can be accessed via four monumental gateways. The western gateway is a waterfront pavilion and the eastern one is the main entrance. The southern and northern gateway is false gates, made for the sake of maintaining symmetry. Every gate is made with red sandstone and has geometric designs. The Bibi ka Makbara is encased with marble only up to the dado level. Above this, it is covered with a fine plaster polished to give a marble-like finish. Only the onion dome was built with marble.

196. The Mughal paintings reached its zenith in the period of Jahangir. He was a naturalist by nature and preferred the paintings of flora and fauna, i.e. birds, animals, trees and flowers. He emphasised on bringing a naturalism to portrait painting. One of the unique trends that developed in this period was of decorated margins around the paintings that were sometimes as elaborate as the paintings themselves. Jahangir was himself considered to be a good artist and he had his own private workshop although no major work by him survives. His atelier mostly created miniature paintings and the most famous amongst them were the naturalistic paintings of the Zebra, the turkey and the cock. One of the most famous artist from his period was Ustad Mansoor who was an expert in drawing the features of the most complex faces. An animal fable called Ayar-i-Danish (Touchstone of Knowledge) was illustrated during his reign.

197. The tomb of Humayun was built by the orders of his first wife and chief consort, Empress Bega Begum. Construction began in 1565 and was completed in 1572. Red Fort Complex constructed in 1639 by the fifth Mughal Emperor Shah Jahan as the palace of his fortified capital Shahjahanabad, the Red Fort is named for its massive enclosing walls of red sandstone. Akbar founded the Fatehpur Sikri city in 1569 after winning Chittor and Ranthambore. Taj Mahal was commissioned in 1632 by the Mughal emperor Shah Jahan to house the tomb of his favourite wife, Mumtaz Mahal.

198. The Ashta Pradhan was designed to encompass all the primary administrative functions of the state, with each minister being given charge of one role in the administration. Ministerial designations were drawn from the Sanskrit language; the eight ministerial roles were as follows:

©Jatin Verma All Rights Reserved. https://www.jatinverma.org P a g e | 90

 Pantpradhan or Peshwa - Prime Minister, general administration of the Empire.  Amatya or Mazumdar - Finance Minister, managing accounts of the Empire.  Shurunavis/Sacheev - Secretary, preparing royal edicts.  Waqianavis - Interior Minister, managing internal affairs especially intelligence and espionage.  Sar-i-Naubat or Senapati - Commander-in-Chief, managing the forces and defence of the Empire.  Sumant/Dabir - Foreign Minister, to manage relationships with other sovereigns.  Nyayadhish - Chief Justice, dispensing justice on civil and criminal matters.  Panditrao - High Priest, managing internal religious matters.

199.  Imad Shahi Dynasty - Berar  Qutub Shahi Dynasty - Golconda  Adil Shahi Dynasty - Bijapur  Nizam shahi - Ahmednagar

200.  Mughal emperor Jahangir granted a Farman to Captain William Hawkins permitting the English to erect a factory at Surat in 1613.

201.  The most important features of the Mughal monuments in India are as follows  The bulbous domes with constricted necks,  The slender minarets with cupolas at the four corners,  Large halls,  Massive vaulted gateways and  Delicate ornamentation,  Ample use of Red Sandstone in Mughal monuments and  Charbagh Style

202.  These are land grants given during the period od different dynasties  Iqta- Delhi sultans  Jagir-Mughals  Amaram-Vijayanagara  Mokasa-Marathas

203. Statement 1 is incorrect. Aurangazeb did not support the Muslim Sultanates of Deccan. His invasions against the were partly due to his hatred of the Shia faith. Statement 2 is incorrect. The destruction of Bijapur and Golconda was a political blunder on the part of Aurangazeb. The barrier between the Mughals and the Marathas was removed and there ensued a direct confrontation between them. Aurangzeb had triumphed but he soon found that the extinction of Bijapur and Golconda was only the beginning of his difficulties. Source) Chapter 17; page number 313

©Jatin Verma All Rights Reserved. https://www.jatinverma.org P a g e | 91

204. Red sandstone was used in Mughal architectures. Pietra dura refers to the technique of creating intricate inlaid pictures from shaped colored stones. This work is prominently visible in TajMahal, Buland Darwaza and Red fort. Charbagh style and Pietra dura ornamentation are the major differences between Sultanate and Mughal architecture styles. Both are characteristic of the Mughal style of architecture. Charbagh style is the presence of a square garden, divided into 4 main parts by causeways in the centre of which ran shallow water channels. Source) Chapter 17; page number 272

205. Statement 1 is incorrect. Akbar was the first Mughal ruler who had the time and means to undertake construction on a large scale. He built a series of forts, the most famous of which is the fort at Agra, built-in red sandstone. Statement 2 is correct. Mosque-building reached its climax under Shah Jahan. The two most noteworthy ones being the Moti Masjid in the built like the Taj entirely in marble, and the other the Jama Masjid at Delhi built in red sandstone. Source) Chapter: 17; page number 267 and 273

206. Daswant and Baswan were two of the famous painters of Akbar's court. Source) Chapter 17; page number 273

207. Statements 1 and 2 are correct. Mughal painting reached a climax under Jahangir who had a very discriminating eye. It was a fashion in the Mughal School for the faces, bodies and feet of the people in a single picture to be painted by different artists. Apart from painting hunting, battle and court scenes, under Jahangir, special progress was made in portrait painting and paintings of animals. Mansure was a great name in this field. Portrait painting also become fashionable. Statement 3 is incorrect. Under Akbar, European painting was introduced at the court by the Portuguese priests. Under their influence, the principles of fore-shortening, whereby near and distant people and things could be placed in perspective were quickly adopted. Source) Chapter 17; page number 276

208. Statement 1 is correct. The withdrawal of royal patronage to painting under Aurangzeb led to the dispersal of artists to different places in the country. This helped in the development of the art of painting in Rajasthan and the Punjab hills giving rise to distinct schools of paintings, for example, Rajasthani and Pahari Schools. Statement 2 is incorrect. Mughal painting was revived under the successors of Aurangzeb, but could never attain the earlier heights. Source) Chapter 17; page number 276

209. Statement 1 is incorrect. The literary activities were not confined to the court and the nobles during the Mughal period. It was essentially a movement of the people, and many scholars and poets of Hindi were found in the countryside and patronized mainly by local landlords and well-to-do public.

©Jatin Verma All Rights Reserved. https://www.jatinverma.org P a g e | 92

Statement 2 is incorrect. During the Mughal period, apart from Persian and Hindi literature, Bengali and Punjabi literature also made strides. It was a golden period for Hindi poetry. Regional languages also developed due to the patronage extended to them by local and regional rulers. The important Hindi poets associated with the Mughal court were Raja Birbal, Man Singh, Bhagwan Das, Narhari etc. Tulsi Das and Surdas were two notable poets who got immortalized owing to their works in Hindi. Regional languages such as Oriya, Rajasthani, and Gujarati had also developed during this period. Many devotional works including the and Mahabharata were translated into regional languages. Source) Chapter 17; page number 276

210. During the Mughal period, some land was converted into Khalisa land from the jurisdiction of the tarafdar. Khalisa land was that piece of land which was used to run the expenses of the king and the royal household. For example, following the annexation of Bijapur and Golconda, the best and most easily manageable jagirs had been kept by Aurangzeb in the khalisa.

©Jatin Verma All Rights Reserved. https://www.jatinverma.org